File Download Area

Information about "Official SAT Practice Test 2007-2008 (2005-2006).pdf"

  • Filesize: 2.20 MB
  • Uploaded: 14/12/2018 20:38:27
  • Status: Active

Free Educational Files Storage. Upload, share and manage your files for free. Upload your spreadsheets, documents, presentations, pdfs, archives and more. Keep them forever on this site, just simply drag and drop your files to begin uploading.

Download Urls

  • File Page Link
    https://www.edufileshare.com/751a5f5a8cc889b4/Official_SAT_Practice_Test_2007-2008_(2005-2006).pdf
  • HTML Code
    <a href="https://www.edufileshare.com/751a5f5a8cc889b4/Official_SAT_Practice_Test_2007-2008_(2005-2006).pdf" target="_blank" title="Download from edufileshare.com">Download Official SAT Practice Test 2007-2008 (2005-2006).pdf from edufileshare.com</a>
  • Forum Code
    [url]https://www.edufileshare.com/751a5f5a8cc889b4/Official_SAT_Practice_Test_2007-2008_(2005-2006).pdf[/url]

[PDF] Official SAT Practice Test 2007-2008 (2005-2006).pdf | Plain Text

SAT Preparation Booklet ™ 2007-08 Get Ready for the SAT ® ➣ Tr y S a m p l e Q u e s t i o n s ➣ Take an Official SAT Practice Test ➣ Go Online for Answer Explanations Find everything in this booklet and more at collegeboard.com/satprep

SAT Preparation Booklet 2 © 2007 The College Board. All rights reserved. College Board, Advanced Placement Program, AP, SAT, and the acorn logo are registered trademarks of the College Board. connect to college success, SAT Reasoning Test, SAT Subject Tests, The Official SAT Study Guide, The Official SAT Question of the Day, The Official SAT Online Course, and SAT Preparation Center are trademarks owned by the College Board. PSAT/NMSQT is a registered trademark of the College Board and National Merit Scholarship Corporation. All other products and services may be trademarks of their respective owners. Visit the College Board on the Web: www.collegeboard.com. Contents SAT Reasoning Test TM . . . . . . . . . . . . . . . . . . . . . . 3 Frequently Asked Questions . . . . . . . . . . . . . . . . . . . . . . . . . . 3 Approaches to Taking the SAT® . . . . . . . . . . . . . . . . . . . . . . . 3 The Critical Reading Section . . . . . . . . . . . . . . . . . 5 Approaches to the Critical Reading Section . . . . . . . . . . . . . 5 Sentence Completions . . . . . . . . . . . . . . . . . . . . . . . . . . . . . . . 5 Passage-Based Reading . . . . . . . . . . . . . . . . . . . . . . . . . . . . . . 6 The Mathematics Section . . . . . . . . . . . . . . . . . . . 13 Calculator Policy . . . . . . . . . . . . . . . . . . . . . . . . . . . . . . . . . . 13 Approaches to the Mathematics Section . . . . . . . . . . . . . . . 13 Mathematics Review . . . . . . . . . . . . . . . . . . . . . . . . . . . . . . . 14 Multiple-Choice Questions . . . . . . . . . . . . . . . . . . . . . . . . . . 20 Student-Produced Response Questions . . . . . . . . . . . . . . . . 23 The Writing Section . . . . . . . . . . . . . . . . . . . . . . . 26 Characteristics of Effective Writing . . . . . . . . . . . . . . . . . . . 26 Improving Sentences . . . . . . . . . . . . . . . . . . . . . . . . . . . . . . . 27 Identifying Sentence Errors . . . . . . . . . . . . . . . . . . . . . . . . . . 28 Improving Paragraphs . . . . . . . . . . . . . . . . . . . . . . . . . . . . . . 29 The Essay. . . . . . . . . . . . . . . . . . . . . . . . . . . . . . . . . . . . . . . . . 31 Scoring the Essay . . . . . . . . . . . . . . . . . . . . . . . . . . . . . . . . . . 36 Scoring Guide . . . . . . . . . . . . . . . . . . . . . . . . . . . . . . . . . . . . . 36 Official SAT Practice Test. . . . . . . . . . . . . . . . . . . 37 About the Practice Test . . . . . . . . . . . . . . . . . . . . . . . . . . . . . 37 Answer Sheet . . . . . . . . . . . . . . . . . . . . . . . . . . . . . . . . . . . . . 38 Official SAT Practice Test . . . . . . . . . . . . . . . . . . . . . . . . . . . 46 Correct Answers and Difficulty Levels . . . . . . . . . . . . . . . . . 84 Scoring the Official SAT Practice Test . . . . . . . . . . . . . . . . . 85 The College Board: Connecting Students to College Success The College Board is a not-for-profit membership asso- ciation whose mission is to connect students to college success and opportunity. Founded in 1900, the association is composed of more than 5,200 schools, colleges, uni- versities, and other educational organizations. Each year, the College Board serves seven million students and their parents, 23,000 high schools, and 3,500 colleges through major programs and services in college admissions, guid- ance, assessment, financial aid, enrollment, and teaching and learning. Among its best-known programs are the SAT®, the PSAT/NMSQT®, and the Advanced Placement Program® (AP®). The College Board is committed to the principles of excellence and equity, and that commitment is embodied in all of its programs, services, activities, and concerns. For further information, visit www.collegeboard.com. , t >˜`ʘÃÜiÀÊ Ý«>˜>̈œ˜Ã iÌÊ>Ê-VœÀiÊ,i«œÀÌ ->“«iÊ ÃÃ>Þà />ŽiÊ Ì…iÊ «À>V̈ViÊ ÌiÃÌÊ ˆ˜Ê Ì…ˆÃÊ LœœŽiÌ]Ê Ì…i˜ÊÊ }œÊ œ˜ˆ˜iÊ ÌœÊ }iÌÊ >˜ÃÜiÀÊ iÝ«>˜>̈œ˜Ã]Ê >ÊÊ «iÀÃœ˜>ˆâi`ÊÃVœÀiÊÀi«œÀÌ]Ê>˜`ÊÃ>“«iÊiÃÃ>ÞÃ°Ê 7…>ÌÊޜսÊw˜`\ ˜ÃÜiÀÊ Ý«>˜>̈œ˜Ã *iÀÃœ˜>ˆâi`Ê -VœÀiÊ,i«œÀÌÊ iÀi½ÃÊ…œÜ\ Ê œÊÌœÊVœi}iLœ>À`°Vœ“ÉÃ>Ì«À>V̈ViÌiÃÌ Ê -ˆ}˜Êˆ˜Ê ­VÀi>ÌiÊ>ÊvÀiiÊ>VVœÕ˜ÌʈvÊÞœÕÊ`œ˜½ÌÊ>Ài>`ÞÊ…>ÛiÊœ˜i® Ê £ Ó

3 SAT Reasoning Test ™ This booklet will answer your questions about the SAT Reasoning Test™ and help you prepare for test day. Frequently Asked Questions Why should I take the SAT ®? Nearly every college and university in the U.S. accepts and uses the SAT as part of its admissions process. In addition, your SAT score can help you get connected to the right col- leges and help you identify opportunities for scholarships. What’s on the SAT? The SAT measures the critical thinking skills that demon- strate how well you analyze and solve problems. The test is composed of three sections: ● Critical reading, which has sentence completion and passage-based reading questions ● Mathematics, which is based on the math that college-bound students typically learn during their first three years of high school ● Writing, which has multiple-choice questions and a written essay How can I prepare for the SAT? ● Take the PSAT/NMSQT® in your junior year. ● Review the sample questions, test-taking approaches, and directions in this booklet. ● Take the official SAT practice test in this booklet and review the answer explanations online. ● Visit the SAT Preparation Center™ at www.collegeboard.com/satprep. How should I get ready for test day? ● Make sure you have on hand all the materials you will need, such as a calculator, No. 2 pencils (no mechanical pencils), a soft eraser, your Admission Ticket, and an official photo ID. ● Check out the route to the test center and know where the weekend entrances are located. ● Get a good night’s sleep. ● Leave yourself plenty of time so you’ll arrive at the test center a little early. How can I help myself feel as confident as possible? ● Think positively. ● Stay focused. ● Concentrate only on what you are doing. ● Keep the test in perspective. ● Remember that you are in control. What do I need to know about the essay? The purpose of the essay is to demonstrate not only how well you write, but also how well you express and back up a point of view. You will have 25 minutes to write your essay, which will count for approximately 30 percent of your writing score. The essay must be written with a No. 2 (soft-lead) pencil and will be scored as a first draft, not a polished piece of writing. Important Information You have 3 hours and 45 minutes to com- plete the entire test. All multiple-choice questions are scored the same way: one point for each correct answer, and one-quarter point subtracted for a wrong answer. No points are sub- tracted for answers left blank. You can always take the test again. One out of every two high school students takes the SAT at least twice. Remember: The SAT is only one factor colleges look at when they consider your application. Make sure you use a No. 2 pencil. It is very important that you fill in the entire circle on the answer sheet darkly and completely. If you change your response, erase it as completely as possible. Approaches to Taking the SAT ● Answer easy questions first. The easier questions are usually at the beginning of the section, and the harder ones are at the end. The exception is in the critical reading section, where questions are ordered according to the logic and organization of each passage. ● Make educated guesses. If you can rule out one or more answer choices for multiple-choice questions, you have a better chance of guessing the right answer. ● Skip questions that you really can’t answer. No points are deducted if an answer is left blank. ● Limit your time on any one question. All ques- tions are worth the same number of points. If you need a lot of time to answer a question, go on to the next one. Later, you may have time to return to the question you skipped. ● Keep track of time. Don’t spend too much time on any group of questions within a section. ● Use your test booklet as scratch paper. ● Mark the questions in your booklet that you skipped and want to return to. ● Check your answer sheet to make sure you are answering the right question. SAT Reasoning Test For daily practice questions, visit The Offi cial SAT Question of the Day ™ at collegeboard.com/ qotd or sign up to receive it by e-mail. Each question has a hint and an answer explanation.

SAT Preparation Booklet 4 The Official SAT Online Course TM • Interactive instruction • 6 offi cial practice tests • Answer explanations • Immediate essay scoring The Official SAT Study Guide TM • 8 offi cial practice tests • Test-taking approaches • Free online score reports • Sample essays and prompts To order, complete Item 17 on the registration form or visit the link below. The Official SAT Question of the Day TM 2008 Calendar • Daily practice questions • Free online answer explanations and hints Get Ready for the with Help from the SAT Test Maker ® Order today! collegeboard.com/satprep

5 The Critical Reading Section The critical reading section gives you a chance to show how well you understand what you read. This section has two types of questions: ● Sentence completions (19 questions) ● Passage-based reading (48 questions) Note: Calculators may not be on your desk or used on the critical reading section of the SAT. Approaches to the Critical Reading Section ● Work on sentence completion questions first. They take less time to answer than the passage-based reading questions. ● The difficulty of sentence completion questions increases as you move through the section. ● Reading questions do not increase in difficulty from easy to hard. Instead, they follow the logic of the passage. ● The information you need to answer each reading question is always in the passage(s). Reading care- fully is the key to finding the correct answer. Don’t be misled by an answer that looks correct but is not supported by the actual text of the passage(s). ● Reading questions often include line numbers to help direct you to the relevant part(s) of the pas- sage. If one word or more is quoted exactly from the passage, the line number(s) where that quota- tion can be found will appear in the test question. You may have to read some of the passage before or after the quoted word(s), however, in order to find support for the best answer to the question. ● Do not jump from passage to passage. Stay with a passage until you have answered as many questions as you can before you proceed to the next passage. ● If you don’t know what a word means in a sentence completion or reading passage, consider related words, familiar sayings and phrases, roots, prefixes, and suffixes. Have you ever heard or seen a word that may be related to it? ● In your test booklet, mark each question you don’t answer so that you can easily go back to it later if you have time. ● Remember that all questions are worth the same number of points regardless of the type or difficulty. Sentence Completions Sentence completion questions measure your ● knowledge of the m eanings of words. ● ability to understand how the different parts of a sentence fit together logically. Directions Each sentence below has one or two blanks, each blank indicating that something has been omitted. Beneath the sentence are five words or sets of words labeled A through E. Choose the word or set of words that, when inserted in the sentence, best fits the meaning of the sentence as a whole. Example: Hoping to ------- the dispute, negotiators proposed a compromise that they felt would be ------- to both labor and management. (A) enforce . . useful (B) end . . divisive (C) overcome . . unattractive (D) extend . . satisfactory (E) resolve . . acceptable a b c d , Answering Sentence Completion Questions One way to answer a sentence completion question with two missing words is to focus first on just one of the two blanks. If one of the words in an answer choice is logically wrong, then you can eliminate the entire choice from consideration. ● Look at the first blank in the above example. Would it make sense to say that “negotiators” who have “proposed a compromise” were hoping to enforce or extend the “dispute”? No, so neither (A) nor (D) can be the correct answer. ● Now you can focus on the second blank. Would the “negotiators” have proposed a compromise that they believed would be divisive or unattractive to “both labor and management”? No, so (B) and (C) can be eliminated, and only choice (E) remains. ● Always check your answer by reading the entire sentence with your choice filled in. Does it make sense to say, “Hoping to resolve the dispute, the negotiators proposed a compromise that they felt would be acceptable to both labor and manage- ment”? Yes. Correct answer: (E) / Difficulty level: Easy The Critical Reading Section

SAT Preparation Booklet 6 Sample Questions 1. Because King Philip’s desire to make Spain the dominant power in sixteenth-century Europe ran counter to Queen Elizabeth’s insistence on autonomy for England, ------- was -------. (A) reconciliation . . assured (B) warfare . . avoidable (C) ruination . . impossible (D) conflict . . inevitable (E) diplomacy . . simple Be sure to look for key words and phrases as you read each sentence. Words such as although, however, if, but, and since are important to notice because they signal how the differ- ent parts of a sentence are logically related to each other. Words such as not and never are important because they indicate negation. In the example above, the entire sentence hinges on a few key words: “Because something ran counter to something else, blank was blank.” ● The word “because” indicates that the information in the first part of the sentence (the part before the comma) explains the reason for the situation described in the second part. The first part states that what King Philip wanted (domination for Spain) “ran counter to” what Queen Elizabeth wanted (independence for England). ● Given that there was such a fundamental disagree- ment between the two monarchs, would reconcilia- tion be assured? Unlikely. ● Would warfare be avoidable? Hardly; warfare might be unavoidable. ● Would ruination be impossible? No. ● Would diplomacy be simple? Not necessarily. ● Only choice (D) fits logically with the key words in the sentence: Because what one person wanted ran counter to what another person wanted, conflict was inevitable. Correct answer: (D) / Difficulty level: Medium 2. There is no doubt that Larry is a genuine -------: he excels at telling stories that fascinate his listeners. (A) braggart (B) dilettante (C) pilferer (D) prevaricator (E) raconteur Some sentence completions contain a colon. This is a signal that the words after the colon define or directly clarify what came before. In this case, “he excels at telling stories that fascinate his listeners” serves to define the word raconteur, choice (E). None of the other words is directly defined by this clause. ● A braggart may or may not excel at telling stories and may actually annoy listeners. ● A dilettante is someone who dabbles at a career or hobby and so may not excel at anything. ● A pilferer steals repeatedly, in small quantities; this has nothing to do with storytelling. ● A prevaricator tells lies, but not necessarily in an accomplished or fascinating way; and the sentence refers to stories, not lies. You should choose the word that best fits the meaning of the sentence as a whole, and only choice (E) does so. Correct answer: (E) / Difficulty level: Hard Passage-Based Reading The reading questions on the SAT measure your ability to read and think carefully about several different passages ranging in length from about 100 to about 850 words. Passages are taken from a variety of fields, including the humanities, social studies, natural sciences, and literary fiction. They vary in style and can include narrative, argu- mentative, and expository elements. Some selections con- sist of a pair of related passages on a shared issue or theme; in some of the questions, you are asked to compare and contrast these passages. The following kinds of questions may be asked about a passage: ● Vocabulary in Context: These questions ask you to determine the meanings of words from their con- text in the reading passage. ● Literal Comprehension: These questions assess your understanding of significant information directly stated in the passage. ● Extended Reasoning: These questions measure your ability to synthesize and analyze information as well as to evaluate the assumptions made and the techniques used by the author. Most of the reading questions fall into this category. You may be asked to identify cause and effect, make infer- ences, recognize a main idea or an author’s tone, and follow the logic of an analogy or an argument. Answering Passage-Based Reading Questions Below are samples of the kinds of reading passages and questions that may appear on your test. For each set of sample materials, ● read the passage carefully. ● decide on the best answer to each question. ● read the explanation for the correct answer.

7 Some of the reading passages in the SAT are as short as a paragraph or two, about 100 words in length. You will also find one or more pairs of related short passages in each edition of the test. Such material will be followed by one to five questions that measure the same kinds of reading skills that are measured by the questions following longer passages. Directions The passages below are followed by questions based on their content; questions following a pair of related passages may also be based on the relationship between the paired passages. Answer the questions on the basis of what is stated or implied in the passages and in any introductory material that may be provided. Sample Questions Questions 3-4 are based on the following passage. “The rock was still wet. The animal was glistening, like it was still swimming,” recalls Hou Xianguang. Hou discovered the unusual fossil while surveying rocks as a paleontology graduate student in 1984, near the Chinese town of Chengjiang. “My teach- ers always talked about the Burgess Shale animals. It looked like one of them. My hands began to shake.” Hou had indeed found a Naraoia like those from Canada. However, Hou’s animal was 15 million years older than its Canadian relatives. Some questions ask you to recognize the meaning of a word as it is used in the passage. 3. In line 4, “surveying” most nearly means (A) calculating the value of (B) examining comprehensively (C) determining the boundaries of (D) polling randomly (E) conducting a statistical study of The word “surveying” has a number of meanings, several of which are included in the choices above. In the context of this passage, however, only (B) makes sense. A student in the field of “paleontology” is one who studies prehistoric life as recorded in fossil remains. One of the activities of a paleontology student would be to examine rocks carefully and “comprehensively” while looking for fossils. ● (A), (C), and (E) are incorrect because someone who studies fossils would not calculate the “value” of rocks, or determine the “boundaries” of rocks, or conduct a “statistical study” of rocks. ● (D) is wrong because “polling” rocks makes no sense at all. Correct answer: (B) / Difficulty level: EasyYou may be asked to make an inference or draw a conclusion about a statement made in the passage. 4. It can be inferred that Hou Xianguang’s “hands began to shake” (line 9) because Hou was (A) afraid that he might lose the fossil (B) worried about the implications of his finding (C) concerned that he might not get credit for his work (D) uncertain about the authenticity of the fossil (E) excited about the magnitude of his discovery In the passage, Hou states that the fossil that he found “looked like” certain other fossils that his “teachers always talked about.” He understands almost immediately, there- fore, the significance of what he has found, and so (E) is the correct answer: Hou’s hands were shaking because he was “excited about the magnitude of his discovery.” ● (A) is wrong because there is no suggestion that Hou was “afraid that he might lose the fossil.” ● (B) and (C) are wrong because the passage does not indicate that Hou was “worried about” his dis- covery or “concerned that he might not get credit.” The passage indicates only that Hou recognized that he had found something valuable. ● (D) is wrong because Hou’s immediate reaction is that he thinks he has found an important fossil. The first two sentences of the passage dramatize the discovery; it is Hou’s excitement, not his uncer- tainty, that causes him to tremble. Correct answer: (E) / Difficulty level: Easy Questions 5-8 are based on the following passage. This passage is adapted from a novel written by a woman in 1899. The novel was banned in many places because of its unconventional point of view. It was eleven o’clock that night when Mr. Pontellier returned from his night out. He was in an excellent humor, in high spir- its, and very talkative. His entrance awoke his wife, who was in bed and fast asleep when he came in. He talked to her while he undressed, telling her anecdotes and bits of news and gossip that he had gathered during the day. She was overcome with sleep, and answered him with little half utterances. He thought it very discouraging that his wife, who was the sole object of his existence, evinced so little interest in things which concerned him and valued so little his conversation. Mr. Pontellier had forgotten the bonbons and peanuts that he had promised the boys. Notwithstanding, he loved them very much and went into the adjoining room where they slept to take a look at them and make Line 5 10 Line 5 10 15 20 The Critical Reading Section

SAT Preparation Booklet 8 An indescribable oppression, which seemed to generate in some unfamiliar part of her consciousness, filled her whole being with a vague anguish. It was like a shadow, like a mist passing across her soul’s summer day. It was strange and unfamiliar; it was a mood. She did not sit there inwardly upbraiding her husband, lamenting at Fate, which had directed her footsteps to the path which they had taken. She was just having a good cry all to herself. The mosquitoes suc- ceeded in dispelling a mood which might have held her there in the darkness half a night longer. The following morning Mr. Pontellier was up in good time to take the carriage which was to convey him to the steamer at the wharf. He was returning to the city to his business, and they would not see him again at the Island till the coming Saturday. He had regained his composure, which seemed to have been somewhat impaired the night before. He was eager to be gone, as he looked forward to a lively week in the financial center. Following are 4 sample questions about this passage. In the actual test, as many as 13 questions may appear with a pas- sage of this length. You may be asked to interpret information presented throughout the passage and to evaluate the effect of the language used by the author. 5. The narrator would most likely describe Mr. Pontellier’s conduct during the evening as (A) typically generous (B) justifiably impatient (C) passionate and irrational (D) patronizing and self-centered (E) concerned and gentle This question asks you to consider a large portion of the passage and to make an inference about the narrator’s view of “Mr. Pontellier’s conduct during the evening.” To answer such a question, you should look carefully at the particular words used and the details mentioned in the passage. For example, in the first paragraph, Mr. Pontellier awakens his wife after his “night out”; he seems not to notice or care that she has been sound asleep. In lines 38–47, the narrator describes Mr. Pontellier speaking to his wife in a superior and condescending manner about “a mother’s place” in caring for children and about how hard he works at “his brokerage business.” ● (A) and (E) are not correct because the narrator does not depict Mr. Pontellier’s words and actions during the evening as “generous” or “gentle.” sure that they were resting comfortably. The result of his investigation was far from satisfactory. He turned and shifted the youngsters about in bed. One of them began to kick and talk about a basket full of crabs. Mr. Pontellier returned to his wife with the information that Raoul had a high fever and needed looking after. Then he lit his cigar and went and sat near the open door to smoke it. Mrs. Pontellier was quite sure Raoul had no fever. He had gone to bed perfectly well, she said, and nothing had ailed him all day. Mr. Pontellier was too well acquainted with fever symptoms to be mistaken. He assured her the child was burning with fever at that moment in the next room. He reproached his wife with her inatten- tion, her habitual neglect of the children. If it was not a mother’s place to look after chil- dren, whose on earth was it? He himself had his hands full with his brokerage business. He could not be in two places at once; mak- ing a living for his family on the street, and staying home to see that no harm befell them. He talked in a monotonous, insistent way. Mrs. Pontellier sprang out of bed and went into the next room. She soon came back and sat on the edge of the bed, leaning her head down on the pillow. She said noth- ing, and refused to answer her husband when he questioned her. When his cigar was smoked out he went to bed, and in half a minute was fast asleep. Mrs. Pontellier was by that time thor- oughly awake. She began to cry a little, and wiped her eyes on the sleeve of her night- gown. She went out on the porch, where she sat down in the wicker chair and began to rock gently to and fro. It was then past midnight. The cottages were all dark. There was no sound abroad except the hooting of an old owl and the everlasting voice of the sea, that broke like a mournful lullaby upon the night. The tears came so fast to Mrs. Pontellier’s eyes that the damp sleeve of her nightgown no longer served to dry them. She went on crying there, not caring any longer to dry her face, her eyes, her arms. She could not have told why she was crying. Such experi- ences as the foregoing were not uncommon in her married life. They seemed never before to have weighed much against the abundance of her husband’s kindness and a uniform devotion which had come to be tacit and self-understood. 80 85 90 95 100 25 30 35 40 45 50 55 60 65 70 75

9 ● (B) is not correct because the narrator does not suggest that Mr. Pontellier’s conduct with his wife is justifiable. ● (C) is not correct; although Mr. Pontellier’s behavior is selfish and inconsiderate, it is not “passionate” —in fact, the narrator states that Mr. Pontellier “talked in a monotonous, insistent way.” ● (D) is correct because it accurately character- izes the narrator’s description of Mr. Pontellier’s behavior during the evening, “patronizing and self-centered.” Someone who is “patronizing” has an attitude of superiority and thus treats others as if they were less important. Correct answer: (D) / Difficulty level: Medium Some questions ask you to focus on a specific piece of infor- mation presented in the passage. 6. In context, the description in lines 46-47 of Mr. Pontellier’s way of speaking suggests the narrator’s belief that his complaints are (A) stumbling and confused (B) familiar and not as urgent as he claims (C) angry and sarcastic (D) too complex to make sense to anyone but himself (E) both rational and thought-provoking In lines 46–47, the narrator describes Mr. Pontellier’s “way of speaking” as “monotonous, insistent.” Previously, Mr. Pontellier had told his wife that one of their sons “had a high fever and needed looking after,” and he had criticized Mrs. Pontellier for her “habitual neglect of the children.” These are seemingly serious matters, and yet Mr. Pontellier is described as not at all excited in the way that he commu- nicates his opinions to his wife. ● (A) is wrong because Mr. Pontellier speaks assert- ively to his wife throughout the passage, not in a “stumbling” or uncertain manner. ● (C) is wrong because statements that are “monoto- nous” and “insistent” are not “angry and sarcastic.” ● (D) and (E) are wrong because the narrator does not indicate that Mr. Pontellier’s statements to his wife are “too complex to make sense” or “rational and thought-provoking.” In fact, the terms “monot- onous” and “insistent” suggest that the statements are rather dull and simpleminded. ● The correct answer is (B) because concerns that are voiced “in a monotonous, insistent way” are likely to be ones that are oft-repeated and “familiar,” and probably “not as urgent” as Mr. Pontellier claims. The statement in lines 53–55 also supports this answer: “When his cigar was smoked out he went to bed, and in half a minute was fast asleep.” Correct answer: (B) / Difficulty level: HardSome questions require you to make an inference or draw a conclusion about what you have read. 7. In lines 56-92, Mrs. Pontellier’s reactions to her husband’s behavior on returning home suggest that (A) she accepts unquestioningly her role of caring for the children (B) this is one of the first times she has acknowledged her unhappiness (C) her marriage is not what is making her so depressed (D) she is angry about something that happened before her husband went out (E) she is not as worldly as her husband is In these lines, Mrs. Pontellier cries for a long time while sitting alone on the porch. Her husband’s treatment of her has upset her greatly. The narrator indicates that such behavior by Mr. Pontellier was “not uncommon” but that Mrs. Pontellier had not previously been too bothered by such incidents: “They seemed never before to have weighed much against the abundance of her husband’s kindness. . . .” ● (A) is not correct because the issue of “caring for the children” is not the focus of this part of the passage; Mrs. Pontellier’s feelings of sadness and “oppression” in this passage are not related to the issue of “her role” as a mother. ● (C) is not correct because it is precisely her rela- tionship with her husband that has made her “so depressed.” ● (D) is not correct because there is no indication in the passage that “something that happened before her husband went out” has made Mrs. Pontellier “angry.” In fact, it is his behavior after his return that upsets her. ● (E) is not correct because whether Mrs. Pontellier is “as worldly as her husband” is irrelevant to her reaction to his treatment of her; the passage sug- gests not that she lacks sophistication, but that he lacks consideration. ● (B) is correct because Mrs. Pontellier’s “strange and unfamiliar” mood of “oppression” and “anguish” marks a new realization on her part of her “unhap- piness” with her husband. Correct answer: (B) / Difficulty level: Medium You may be asked to consider the overall description of a character, event, or phenomenon across an entire passage. 8. The passage shows Mr. Pontellier as happiest when he (A) is attending to his children (B) sits outside and smokes a cigar (C) makes up with his wife after an argument (D) has been away from home or is about to leave home (E) has showered his children with gifts of candy The Critical Reading Section

SAT Preparation Booklet 10 The passage begins with Mr. Pontellier “in an excellent humor,” having just returned after a night away from home. He becomes less happy, however, when his wife is too sleepy to talk with him, and when he discovers that his son Raoul “had a high fever and needed looking after.” Subsequently, he lectures his wife about their family roles and responsibilities, finishes his cigar, and goes to bed. The next morning, Mr. Pontellier has “regained his composure” and is “eager to be gone, as he looked forward to a lively week” away from his family at work. ● (A) and (E) are not correct because Mr. Pontellier gets upset the one time that he is “attending to” his sons, and he has forgotten to bring them the treats that he had promised. ● (B) is not correct because Mr. Pontellier is described as neither happy nor unhappy while he smokes; there are other occasions in the passage when he is happier. ● (C) is not correct because the passage never shows Mr. Pontellier making up with his wife after their argument. ● (D) is the correct answer based on the description of a happy Mr. Pontellier at the beginning and the end of the passage, when “he has been away from home or is about to leave home.” Correct answer: (D) / Difficulty level: Medium Questions 9-12 are based on the following passages. These two passages were adapted from autobiographical works. In the first, a playwright describes his first visit to a theater in the 1930’s; in the second, an eighteenth-century writer describes two visits to theaters in London. Passage 1 I experienced a shock when I saw a cur- tain go up for the first time. My mother had taken me to see a play at the Schubert Theater on Lenox Avenue in Harlem in New York City. Here were living people talking to one another inside a large ship whose deck actually heaved up and down with the swells of the sea. By this time I had been going to the movies every Saturday afternoon —Charlie Chaplin’s little comedies, adven- ture serials, Westerns. Yet once you knew how they worked, movies, unlike the stage, left the mind’s grasp of reality intact since the happenings were not in the theater where you sat. But to see the deck of the ship in the theater moving up and down, and people appearing at the top of a ladder or disappearing through a door—where did they come from and where did they go? Obviously into and out of the real world of Lenox Avenue. This was alarming. And so I learned that there were two kinds of reality, but that the stage was far more real. As the play’s melodramatic story developed, I began to feel anxious, for there was a villain on board who had a bomb and intended to blow everybody up. All over the stage people were looking for him but he appeared, furtive and silent, only when the searchers were facing the other way. They looked for him behind posts and boxes and on top of beams, even after the audience had seen him jump into a barrel and pull the lid over him. People were yelling, “He’s in the barrel,” but the passengers were deaf. What anguish! The bomb would go off any minute, and I kept clawing at my mother’s arm, at the same time glancing at the the- ater’s walls to make sure that the whole thing was not really real. The villain was finally caught, and we happily walked out onto sunny Lenox Avenue, saved again. Passage 2 I was six years old when I saw my first play at the Old Drury. Upon entering the theater, the first thing I beheld was the green curtain that veiled a heaven to my imagina- tion. What breathless anticipations I endured! I had seen something like it in an edition of Shakespeare, an illustration of the tent scene with Diomede in Troilus and Cressida. (A sight of that image can always bring back in a measure the feeling of that evening.) The balconies at that time, full of well-dressed men and women, projected over the orchestra pit; and the pilasters* reaching down were adorned with a glister- ing substance resembling sugar candy. The orchestra lights at length rose. Once the bell sounded. It was to ring out yet once again— and, incapable of the anticipation, I reposed my shut eyes in a sort of resignation upon my mother’s lap. It rang the second time. The curtain drew up—and the play was Artaxerxes! Here was the court of ancient Persia. I took no proper interest in the action going on, for I understood not its import. Instead, all my feeling was absorbed in vision. Gorgeous costumes, gardens, palaces, princesses, passed before me. It was all enchantment and a dream. After the intervention of six or seven years I again entered the doors of a theater. That old Artaxerxes evening had never done ringing in my fancy. I expected the same feelings to come again with the same occa- sion. But we differ from ourselves less at sixty and sixteen, than the latter does from six. In that interval what had I not lost! At six I knew nothing, understood nothing, discriminated nothing. I felt all, loved all, Line 5 10 15 2045 50 55 60 65 70 75 8025 30 35 40

wondered all. I could not tell how, but I had left the temple a devotee, and was returned a rationalist. The same things were there materially; but the emblem, the reference, was gone. The green curtain was no longer a veil, drawn between two worlds, the unfold- ing of which was to bring back past ages, but a certain quantity of green material, which was to separate the audience for a given time from certain of their fellows who were to come forward and pretend those parts. The lights—the orchestra lights—came up a clumsy machinery. The first ring, and the second ring, was now but a trick of the prompter’s bell. The actors were men and women painted. I thought the fault was in them; but it was in myself, and the alteration which those many centuries—those six short years—had wrought in me. * Pilasters are ornamental columns set into walls. Following are four sample questions about this pair of related passages. In the test, some questions will focus on Passage 1, others will focus on Passage 2, and about half or more of the questions following each pair of passages will focus on the relationships between the passages. Some questions require you to identify shared ideas or simi- larities between the two related passages. 9. The authors of both passages describe (A) a young person’s sense of wonder at first seeing a play (B) a young person’s desire to become a playwright (C) the similarities between plays and other art forms (D) how one’s perception of the theater may develop over time (E) the experience of reading a play and then seeing it performed To answer this question, you have to figure out what these two passages have in common. The subject of Passage 1 is a child’s first visit to see a play performed in a theater, and how captivated he was by the entire experience. Passage 2 describes two different visits to the theater; at age six the child is entranced by the spectacle of the performance but, “after the intervention of six or seven years,” the older and now more knowledgeable child is not so impressed. (A) is the correct answer because all of Passage 1 and the first half of Passage 2 describe “a young person’s sense of won- der at first seeing a play.” ● (B) is wrong; even though the introduction to these passages reveals that one of the authors is a “playwright,” there is no mention in either passage of a “desire to become a playwright.” ● (C) is wrong because Passage 1 mentions differ- ences rather than “similarities” between plays and movies, and Passage 2 does not mention any “other art forms” at all. ● (D) is wrong because only Passage 2 discusses “how one’s perception of the theater may develop over time”—this subject is unmentioned in Passage 1. ● (E) is wrong because there is no reference in either passage to “the experience of reading a play.” Correct answer: (A) / Difficulty level: Easy Some questions assess your comprehension of information that is directly stated in a passage. 10. The “happenings” mentioned in line 14 refer to the (A) work undertaken to produce a movie (B) events occurring in the street outside the theater (C) fantasies imagined by a child (D) activity captured on the movie screen (E) story unfolding on the stage To answer this question correctly, you have to understand lines 11–15, a rather complex sentence that makes an important distinction in Passage 1. The author indicates that, unlike plays, movies leave “the mind’s grasp of reality intact,” because the “happenings” in a movie are not occur- ring in the actual theater. Instead, images are projected on a screen in the theater. Thus (D) is the correct answer; the word “happenings” refers to the “activity captured on the movie screen.” ● (A) and (B) are wrong because, when you insert them in place of the word “happenings,” the sen- tence in lines 11–15 makes no sense. ● (C) is wrong; even if the movies being referred to include “fantasies” in them, they are not “imagined by a child” but are actually projected on the movie screen. ● (E) is wrong because, in line 14, “happenings” refers to the “story unfolding” in a movie, not “on the stage.” Correct answer: (D) / Difficulty level: Medium You may be asked to recognize the author’s tone or attitude in a particular part of a passage, or in the passage as a whole. 11. In the final sentence of Passage 2 (“I thought . . . in me”), the author expresses (A) exultation (B) vindication (C) pleasure (D) regret (E) guilt Even though this question focuses on a single sentence, you must understand the context in which the statement occurs in order to determine the feeling expressed by the author. In the second paragraph of Passage 2, the author states that the experience of attending a play at age 12 or 13 was much different than at age 6. “The same things were there materially” in the theater, but the older child knew much more than the younger one about what was going 85 90 95 The Critical Reading Section 11

SAT Preparation Booklet 12 on. Ironically, this increased knowledge actually decreased the author’s pleasure in attending the play. “In that interval what had I not lost!” the author exclaims in line 78. Where the younger child saw nobles in “the court of ancient Persia,” the older child saw “men and women painted.” Thus the final sentence of Passage 2 expresses “regret” con- cerning the changes that “those many centuries—those six short years—had wrought” in the author. (D) is the correct answer. ● (A) and (C) are incorrect because the author does not feel “exultation” about or take “pleasure” in the “alteration” that has occurred; on the contrary, the author laments it. ● (B) is incorrect because there is no expression of “vindication” in the final sentence; the author is not trying to justify, support, or defend the expe- riences described in the passage, but rather to explain the changes that have occurred due to the passing of time. ● (E) is incorrect because, even though the final sen- tence states that the “fault” was not in the actors but in the now more knowledgeable child, the author feels no “guilt” about the change. There is no way to avoid the passage of time (and the learn- ing that goes along with it). Aging is not the child’s “fault,” but the loss of a youthful sense of wonder and innocence can still cause regret. Correct answer: (D) / Difficulty level: Hard Some questions require you to determine and compare the primary purpose or main idea expressed in each passage. 12. Which of the following best describes the difference between Passages 1 and 2 ? (A) Passage 1 remembers an event with fondness, while Passage 2 recalls a similar event with bitter detachment. (B) Passage 1 considers why the author responded to the visit as he did, while Passage 2 supplies the author’s reactions without further analysis. (C) Passage 1 relates a story from a number of different perspectives, while Passage 2 maintains a single point of view. (D) Passage 1 treats the visit to the theater as a disturbing episode in the author’s life, while Passage 2 describes the author’s visit as joyful. (E) Passage 1 recounts a childhood experience, while Passage 2 examines how a similar experience changed over time. This question asks you to do two things: first, understand the overall subject or purpose of each passage; second, recognize an important “difference between” the two. The correct answer is (E) because the entire first passage does indeed tell the story of a particular “childhood experi- ence”—a trip to the theater—whereas the second passage describes two different trips to the theater and how the “experience changed over time.” ● (A) is wrong because there is neither bitterness nor “detachment” in Passage 2. In fact, the first paragraph of Passage 2 expresses excitement and “enchantment,” and the second paragraph expresses disappointment and regret. ● (B) is wrong because Passage 2 includes a great deal more than just “the author’s reactions” to visiting the theater; most of the second paragraph provides “further analysis” of what had changed and why the reactions to the two visits were so different. ● (C) is wrong because it reverses the two narrative approaches in this pair of passages. Passage 1 “maintains a single point of view,” that of the youthful first-time theatergoer, whereas the author of Passage 2 presents at least two “different perspectives,” that of the enchanted six-year-old and of the older child returning to the theater. ● (D) is wrong because the author of Passage 1 does not find his first visit to the theater “disturbing” in a negative way. Although he feels “shock” when the curtain goes up and anxiety during the play, these responses merely indicate how effective and “real” the performance was for him. In the end, the child and his mother walked “happily” out of the theater. Correct answer: (E) / Difficulty level: Easy

The Mathematics Section 13 The Mathematics Section The mathematics section of the SAT contains two types of questions: ● Standard multiple-choice (44 questions) ● Student-produced response questions that provide no answer choices (10 questions) Some questions are like questions you may have seen in your mathematics courses. The ability to reason logically in a variety of situations, some of which may be new to you, is tested throughout. Calculator Policy We recommend that you bring a calculator to use on the mathematics section of the SAT. Every question on the test can be solved without a calculator; however, using a calcula- tor on some questions may be helpful to you. A scientific or graphing calculator is recommended. Acceptable Calculators Calculators permitted during testing are: ● Graphing calculators ● Scientific calculators ● Four-function calculators (not recommended) If you have a calculator with characters that are 1 inch or higher, or if your calculator has a raised display that might be visible to other test-takers, you will be seated at the dis- cretion of the test supervisor. You will not be allowed to share calculators. You will be dismissed and your scores will be canceled if you use your calculator to share information during the test or to remove test questions or answers from the test room. Calculator Tips ● Remember to bring your calculator to the test. Calculators will not be available at the test center. You should be familiar with how to use the calculator you bring to the test. ● Make sure your calculator is in good working order and that batteries are fresh. If your calculator fails during testing and you have no backup, you’ll have to complete the test without it. ● Don’t buy an expensive, sophisticated calculator just to take the test. Although you can use them for the test, more sophisticated calculators are not required for any problem. ● Don’t try to use a calculator on every question. First, decide how you will solve the problem, and then decide whether to use the calculator. The calcu- lator is meant to aid you in problem solving, not to get in the way. ● Get your thoughts down before using your calcula- tor. It may help to do scratchwork in the test book. ● Take the practice test in this booklet with a calcula- tor at hand. This will help you determine how much you will probably use a calculator the day of the test. Unacceptable Calculators Unacceptable calculators are those that: ● use QWERTY (typewriter-like) keypads ● require an electrical outlet ● “talk” or make unusual noises ● use paper tape ● are electronic writing pads, pen input/stylus-driven devices, pocket organizers, cell phones, powerbooks, or handheld or laptop computers Approaches to the Mathematics Section ● Familiarize yourself with the directions ahead of time. ● The test does not require you to memorize for- mulas. Commonly used formulas are provided in the test book at the beginning of each mathematics section. It is up to you to decide which formula is appropriate. ● Read the problem carefully. Note key words that tell you what the problem is asking. Ask yourself the following questions before you solve each problem: What is the question asking? What do I know? ● With some problems, it may be useful to draw a sketch or diagram of the given information. ● Use the test book for scratchwork. You are not expected to do all the reasoning and figuring in your head. You will not receive credit for anything writ- ten in the booklet, but you will be able to check your work easily later. ● Decide when to use a calculator. ● For multiple-choice questions, you may want to refer to the answer choices before you determine your answer. ● Eliminate choices. If you don’t know the correct answer to a question, try some of the choices. It’s sometimes easier to find the wrong answers than the correct one. On some questions, you can eliminate all the incorrect choices. ● Make sure your answer is a reasonable answer to the question asked. This is especially true for student-produced response questions, where no answer choices are given. ● All figures are drawn to scale unless otherwise indicated.

SAT Preparation Booklet 14 Mathematics Review Number and Operations (20–25%) ● Arithmetic word problems (including percent, ratio, and proportion) ● Properties of integers (even, odd, prime numbers, divisibility, etc.) ● Rational numbers ● Sets (union, intersection, elements) ● Counting techniques ● Sequences and series (including exponential growth) ● Elementary number theory Algebra and Functions (35–40%) ● Substitution and simplifying algebraic expressions ● Properties of exponents ● Algebraic word problems ● Solutions of linear equations and inequalities ● Systems of equations and inequalities ● Quadratic equations ● Rational and radical equations ● Equations of lines ● Absolute value ● Direct and inverse variation ● Concepts of algebraic functions ● Newly defined symbols based on commonly used operations Geometry and Measurement (25–30%) ● Area and perimeter of a polygon ● Area and circumference of a circle ● Volume of a box, cube, and cylinder ● Pythagorean Theorem and special properties of isosceles, equilateral, and right triangles ● Properties of parallel and perpendicular lines ● Coordinate geometry ● Geometric visualization ● Slope ● Similarity ● Transformations Data Analysis, Statistics, and Probability (10–15%) ● Data interpretation (tables and graphs) ● Descriptive statistics (mean, median, and mode) ● Probability Number and Operations ● Integers: . . . , −4, −3, −2, −1, 0, 1, 2, 3, 4, . . . (Note: zero is neither positive nor negative.) ● Consecutive Integers: Integers that follow in sequence; for example, 22, 23, 24, 25. Consecutive integers can be more generally represented by n, n + 1, n + 2, n + 3, . . . ● Odd Integers: . . . , −7, −5, −3, −1, 1, 3, 5, 7, . . . , 21k+ , . . . where k is an integer ● Even Integers: . . . , −6, −4, −2, 0, 2, 4, 6, . . . , 2k , . . . , where k is an integer (Note: zero is an even integer.) ● Prime Numbers: 2, 3, 5, 7, 11, 13, 17, 19, . . . (Note: 1 is not a prime and 2 is the only even prime.) ● Digits: 0, 1, 2, 3, 4, 5, 6, 7, 8, 9 (Note: the units digit and the ones digit refer to the same digit in a number. For example, in the number 125, the 5 is called the units digit or the ones digit.) Percent Percent means hundredths, or number out of 100. For example, 40 percent means 40 100 or 0.40 or 2 5. Problem 1: If the sales tax on a $30.00 item is $1.80, what is the sales tax rate? Solution: $. $ .180 10030 00 =×n n=66,%so is the sales tax rate. Percent Increase / Decrease Problem 2: If the price of a computer was decreased from $1,000 to $750, by what percent was the price decreased? Solution: The price decrease is $250. The percent decrease is the value of n in the equation 250 1 000, = n 100 . The value of n is 25, so the price was decreased by 25%. Note: n% increase means increase original = n 100 ; n% decrease means decrease ori ginal = n 100 .

The Mathematics Section 15 Average Speed Problem: José traveled for 2 hours at a rate of 70 kilome- ters per hour and for 5 hours at a rate of 60 kilometers per hour. What was his average speed for the 7-hour period? Solution: In this situation, the average speed was total distance total time The total distance was 2 hr 70km hr ⎛ ⎝ ⎜⎞ ⎠ ⎟ + 5 hr 60km hr ⎛ ⎝ ⎜⎞ ⎠ ⎟ = 440 km. The total time was 7 hours. Thus, the average speed was 440 7km hr = 626 7 kilometers per hour. Note: In this example, the average speed over the 7-hour period is not the average of the two given speeds, which would be 65 kilometers per hour. Sequences Two common types of sequences that appear on the SAT are arithmetic and geometric sequences. An arithmetic sequence is a sequence in which successive terms differ by the same constant amount. For example: 3, 5, 7, 9, . . . is an arithmetic sequence. A geometric sequence is a sequence in which the ratio of successive terms is a constant. For example: 2, 4, 8, 16, . . . is a geometric sequence. A sequence may also be defined using previously defined terms. For example, the first term of a sequence is 2, and each successive term is 1 less than twice the preceding term. This sequence would be 2, 3, 5, 9, 17, . . . On the SAT, explicit rules are given for each sequence. For example, in the sequence above, you would not be expect- ed to know that the 6th term is 33 without being given the fact that each term is one less than twice the preced- ing term. For sequences on the SAT, the first term is never referred to as the zeroth term. Algebra and Functions Factoring You may need to apply these types of factoring: xxxx2 22 += + () xxx2 111 −= + () − () xx x x x22 21 1 1 1 ++=+ () + () =+() 25321 3 2xx x x+−= − () + () Functions A function is a relation in which each element of the domain is paired with exactly one element of the range. On the SAT, unless otherwise specified, the domain of any function f is assumed to be the set of all real numbers x for which fx() is a real number. For example, if fx x() =+2 , the domain of f is all real numbers greater than or equal to −2 . For this function, 14 is paired with 4, since f14 14 2 16 4() =+= = . Note: the symbol represents the positive, or principal, square root. For example, 16 4= , not ± 4. Exponents You should be familiar with the following rules for exponents on the SAT. For all values of abx y,,, : xx xab ab⋅ = + xxab ab() = ⋅ xy x y a aa () = ⋅ For all values of abx y,, , :>>00 x xxa bab = − x yx y a a a ⎛ ⎝ ⎜⎞ ⎠ ⎟= x xa a −=1 Also, xx a ba b = . For example, xx 2 32 3 = . Note: For any nonzero number x , it is true that x0 1 =. Variation Direct Variation: The variable y is directly proportional to the variable x if there exists a nonzero constant k such that ykx= . Inverse Variation: The variable y is inversely proportional to the variable x if there exists a nonzero constant k such that yk xxy k ==or . Absolute Value The absolute value of x is defined as the distance from x to zero on the number line. The absolute value of x is written as x. For all real numbers x: xxx xx =≥ −< ⎧ ⎨ ⎩, ,if if0 0 For example: 22 20 222 20 00=> −=−− = − = ⎧ ⎨ ⎪ ⎩ ⎪, () ,since since<

SAT Preparation Booklet 16 Geometry and Measurement Figures that accompany problems are intended to provide information useful in solving the problems. They are drawn as accurately as possible EXCEPT when it is stated in a particular problem that the figure is not drawn to scale. In general, even when figures are not drawn to scale, the rela- tive positions of points and angles may be assumed to be in the order shown. Also, line segments that extend through points and appear to lie on the same line may be assumed to be on the same line. A point that appears to lie on a line or curve may be assumed to lie on the line or curve. The text “Note: Figure not drawn to scale” is included with the figure when degree measures may not be accurately shown and specific lengths may not be drawn proportion- ally. The following examples illustrate what information can and cannot be assumed from figures. Example 1: Since AD and BE are line segments, angles ACB and DC E are vertical angles. Therefore, you can conclude that xy= . Even though the figure is drawn to scale, you should NOT make any other assumptions without additional informa- tion. For example, you should NOT assume that AC CD= or that the angle at vertex E is a right angle even though they might look that way in the figure. Example 2: Note: Figure not drawn to scale. A question may refer to a triangle such as ABC above. Although the note indicates that the figure is not drawn to scale, you may assume the following from the figure: ● ABD and DBC are triangles. ● D is between A and C. ● A, D, and C are points on a line. ● The length of AD is less than the length of AC . ● The measure of angle ABD is less than the measure of angle AB C. You may not assume the following from the figure: ● The length of AD is less than the length of DC . ● The measures of angles BAD and BDA are equal. ● The measure of angle ABD is greater than the measure of angle DBC . ● Angle ABC is a right angle. Properties of Parallel Lines  m k a°b° c°d° w°x° y°z° 1. If two parallel lines are cut by a third line, the alternate interior angles are congruent. In the figure above, cx wd==and 2. If two parallel lines are cut by a third line, the cor- responding angles are congruent. In the figure, awbxcy d z=== =,,,and 3. If two parallel lines are cut by a third line, the sum of the measures of the interior angles on the same side of the transversal is 180°. In the figure, cw d x+= +=180 180and Angle Relationships 60°50° x° y° z° 1. The sum of the measures of the interior angles of a triangle is 180°. In the figure above, x=70 because 60 50 180++=x 2. When two lines intersect, vertical angles are congruent. In the figure, y=50 3. A straight angle measures 180°. In the figure, z=130 because z+=50 180

The Mathematics Section 17 4. In any triangle, the longest side is opposite the largest angle, and the shortest side is opposite the smallest angle. In the figure below, abc

SAT Preparation Booklet 18 Volume Volume of a rectangular solid (or cube) = ××wh ( is the length, w is the width, and h is the height) Volume of a right circular cylinder = πrh2 (r is the radius of the base, and h is the height) Be familiar with the formulas that are provided in the Reference Information included with the test directions. Refer to the test directions in the sample test in this publication. Coordinate Geometry 1. In questions that involve the x- and y-axes, x-values to the right of the y-axis are positive and x-values to the left of the y-axis are negative. Similarly, y-values above the x-axis are positive and y-values below the x-axis are negative. In an ordered pair (, )xy , the x-coordinate is written first. Point P in the figure above appears to lie at the intersection of gridlines. From the figure, you can conclude that the x-coordinate of P is −2 and the y-coordinate of P is 3. Therefore, the coordinates of point P are () , −23 . Similarly, you can conclude that the line shown in the figure passes through the point with coordinates () , −−21 and the point (), 22 . 2. Slope of a line =change in -coordinates change in -coordiny x a ates Slope ofPQ==4 22 Slope ofl=−− −−=− 12 223 4 () A line that slopes upward as you go from left to right has a positive slope. A line that slopes down- ward as you go from left to right has a negative slope. A horizontal line has a slope of zero. The slope of a vertical line is undefined. Parallel lines have the same slope. The product of the slopes of two perpendicular lines is −1, provided the slope of each of the lines is defined. For example, any line perpendicular to line above has a slope of 4 3. The equation of a line can be expressed as ymxb=+ , where m is the slope and b is the y-intercept. Since the slope of line is −3 4 , the equation of line can be expressed as yxb=− +3 4 . Since the point () , −21 is on the line, xy=− =21and must satisfy the equa- tion. Hence, 13 21 2 =+ =−bb,so , and the equation of line is yx=− −3 41 2 . 3. A quadratic function can be expressed as yaxh k=− () +2 where the vertex of the parabola is at the point (, )hk and a≠0 . If a>0 , the parabola opens upward; and if a

The Mathematics Section 19 Data Analysis, Statistics, and Probability Measures of Center An average is a statistic that is used to summarize data. The most common type of average is the arithmetic mean. The average (arithmetic mean) of a list of n numbers is equal to the sum of the numbers divided by n. For example, the mean of 2, 3, 5, 7, and 13 is equal to 235713 56 +++ + = . When the average of a list of n numbers is given, the sum of the numbers can be found. For example, if the average of six numbers is 12, the sum of these six numbers is The median of a list of numbers is the number in the mid- dle when the numbers are ordered from greatest to least or from least to greatest. For example, the median of 3, 8, 2, 6, and 9 is 6 because when the numbers are ordered, 2, 3, 6, 8, 9, the number in the middle is 6. When there is an even number of values, the median is the same as the mean of the two middle numbers. For example, the medi- an of 6, 8, 9, 13, 14, and 16 is the mean of 9 and 13, which is 11. The mode of a list of numbers is the number that occurs most often in the list. For example, 7 is the mode of 2, 7, 5, 8, 7, and 12. The list 2, 4, 2, 8, 2, 4, 7, 4, 9, and 11 has two modes, 2 and 4. Note: On the SAT, the use of the word average refers to the arithmetic mean and is indicated by “average (arithmetic mean).” An exception is when a question involves average rate (see page 15). Questions involving median and mode will have those terms stated as part of the question’s text. Probability Probability refers to the chance that a specific outcome can occur. When outcomes are equally likely, probability can be found by using the following definition: number of ways that a specific outcome can occur total number of possible outcomes For example, if a jar contains 13 red marbles and 7 green marbles, the probability that a marble selected from the jar at random will be green is If a particular outcome can never occur, its probability is 0. If an outcome is certain to occur, its probability is 1. In general, if p is the probability that a specific out- come will occur, values of p fall in the range 01≤≤p . Probability may be expressed as either a decimal, a fraction, or a ratio.

SAT Preparation Booklet 20 out techniques you’ll be able to use again. Most problems can be solved in a variety of ways, so don’t be concerned if your method is different from the one given. Note that the directions indicate that you are to select the best of the choices given. The questions that follow will give you an idea of the type of mathematical thinking required to solve problems on the SAT. First, try to answer each question yourself, and then read the solutions that follow. These solutions may give you new insights into solving the problems or point Directions Sample Questions Below are seven examples of standard multiple-choice questions. Following each question, you will find one or two solutions. 1. A special lottery is to be held to select the student who will live in the only deluxe room in a dormi- tory. There are 100 seniors, 150 juniors, and 200 sophomores who applied. Each senior’s name is placed in the lottery 3 times; each junior’s name, 2 times; and each sophomore’s name, 1 time. If a student's name is chosen at random from the names in the lottery, what is the probability that a senior's name will be chosen? (A) 1 8 (B) 2 9 (C) 2 7 (D) 3 8 (E) 1 2 Correct answer: (D) / Difficulty level: Medium Notes 1. The use of a calculator is permitted. 2. All numbers used are real numbers. 3. Figures that accompany problems in this test are intended to provide information useful in solving the problems. They are drawn as accurately as possible EXCEPT when it is stated in a specific problem that the figure is not drawn to scale. All figures lie in a plane unless otherwise indicated. 4. Unless otherwise specified, the domain of any function f is assumed to be the set of all real numbers x for which f(x) is a real number. h r r h b A = r 2 C = 2 r A = bh V = wh V = r 2h The number of degrees of arc in a circle is 360. The sum of the measures in degrees of the angles of a triangle is 180. b ac c2 = a 2 + b 2 Special Right Triangles x 3 2x x 60° 30° s s45° 45° A = w w wh Reference Information 2 s 1 2 For this section, solve each problem and decide which is the best of the choices given. Fill in the corresponding circle on the answer sheet. You may use any available space for scratchwork. To determine the probability that a senior’s name will be chosen, you must determine the total number of seniors’ names that are in the lottery and divide this number by the total number of names in the lottery. Since each senior’s name is placed in the lottery 3 times, there are 3 100 300×= seniors’ names. Likewise, there are 21 50 300 ×= juniors’ names and 1 200 200×= sophomores’ names in the lottery. The probability that a senior’s name will be chosen is 300 300 300 200300 8003 8 ++==. Multiple-Choice Questions

The Mathematics Section 21 NOONTIME TEMPERATURES IN HILO, HAWAII Mon Tue Wed Thu Fri Sat Sun 66 78 75 69 78 77 70 2. The table above shows the temperatures at noon, in degrees Fahrenheit, in a city in Hawaii over a one- week period. If m represents the median of these tem- peratures, f represents the temperature that occurred most often, and a represents the average (arithmetic mean) of these seven temperatures, which of the following is the correct order of m, f, and a ? (A) am f

SAT Preparation Booklet 22 Note: Figure not drawn to scale. ab c 5. If two sides of the triangle above have lengths 5 and 6, the perimeter of the triangle could be which of the following? I. 15 II. 20 III. 22 (A) I only (B) I and II only (C) I and III only (D) II and III only (E) I, II, and III Correct answer: (B) / Difficulty level: Hard In questions of this type, statements I, II, and III should each be considered independently of the others. In this question, you must determine which of those statements could be true. ● Statement I states that 15 could be the perimeter of the triangle. This is true. If the perimeter of the triangle is 15, and two sides have lengths 5 and 6, then the third side of the triangle would have length 15 − (6 + 5), or 4. A triangle can have side lengths of 4, 5, and 6. So the perimeter of the triangle could be 15. ● Similarly, statement II is true. If 20 is the perimeter of the triangle, then the third side of the triangle would have length 20 – (6 + 5), or 9. A triangle can have side lengths of 5, 6, and 9. So the perimeter of the triangle could be 20. ● Finally, consider whether the triangle could have a perimeter of 22. In this case, the length of the third side would be 22 – (6 + 5) = 11. By the Triangle Inequality, the sum of the lengths of any two sides of a triangle must be greater than the length of the third side. Since the sum of 5 and 6 is not greater than 11, it follows that 5, 6, and 11 cannot be the lengths of the sides of a triangle, and so the given triangle cannot have a perimeter of 22. Therefore, the correct answer to the question is I and II only, which is choice (B). 6. If xx x xm >=1 3 and , what is the value of m ? (A) −7 2 (B) −3 (C) −5 2 (D) −2 (E) −3 2 Correct answer: (C) / Difficulty level: Medium Since x can be written as x 1 2 and 13x can be written as x−3, the left side of the equation is xx x x x x m 1 21 25 25 2 33 ⋅= = = −− ⎛ ⎝ ⎜⎞ ⎠ ⎟ −−.Since , the value of m is −5 2 . 7. If k is divisible by 2, 3, and 15, which of the follow- ing is also divisible by these numbers? (A) k+5 (B) k+15 (C) k+20 (D) k+30 (E) k+45 Correct answer: (D) / Difficulty level: Medium Since k is divisible by 2, 3, and 15, k must be a multiple of 30, as 30 is the least common multiple of 2, 3, and 15. Some multiples of 30 are 0, 30, 60, 90, and 120. ● If you add two multiples of 30, the sum will also be a multiple of 30. For example, 60 and 90 are multiples of 30 and their sum, 150, is also a multiple of 30. ● If you add a multiple of 30 to a number that is not a multiple of 30, the sum will not be a multiple of 30. For example, 60 is a multiple of 30 and 45 is not. Their sum, 105, is not a multiple of 30. ● The question asks which answer choice is divis- ible by 2, 3, and 15—that is, which answer choice is a multiple of 30. All the answer choices are in the form of “k plus a number.” Only choice (D), k+30 , is the sum of k and a multiple of 30. The sum of k and 30 is also a multiple of 30, so the correct answer is choice (D).

The Mathematics Section 23 Student-Produced Response Questions Questions of this type have no answer choices provided. Instead, you must solve the problem and fill in your answer on a special grid. Ten questions on the test will be of this type. It is very important for you to understand the directions for entering answers on the grid. You will lose valuable testing time if you read the directions for the first time when you take the test. A primary advantage of this format is that it allows you to enter the form of the answer that you obtain, whether whole number, decimal, or fraction. For example, if you obtain 2/5, you can grid 2/5. If you obtain .4, you can grid .4. Generally, you should grid the form of the answer that you obtain naturally in solving the problem. The grid will only hold numbers that range from 0 to 9999. Decimals and fractions can also be gridded. Each of the remaining questions requires you to solve the problem and enter your answer by marking the circles in the special grid, as shown in the examples below. You may use any available space for scratchwork. Decimal Answers: If you obtain a decimal answer with more digits than the grid can accommodate, it may be either rounded or truncated, but it must fill the entire grid. For example, if you obtain an answer such as 0.6666..., you should record your result as .666 or .667. A less accurate value such as .66 or .67 will be scored as incorrect. Acceptable ways to grid are: 2 3 Note: You may start your answers in any column, space permitting. Columns not needed should be left blank. Mark no more than one circle in any column. Because the answer sheet will be machine- scored, you will receive credit only if the circles are filled in correctly. Although not required, it is suggested that you write your answer in the boxes at the top of the columns to help you fill in the circles accurately. Some problems may have more than one correct answer. In such cases, grid only one answer. No question has a negative answer. Mixed numbers such as 3 must be gridded as 3.5 or 7 2. (If is gridded, it will be interpreted as , not 3 .) 1 2 1 2 31 2 • • • • • • 1 2 3 4 5 60 1 2 3 4 5 60 1 2 3 4 5 60 1 2 3 4 5 61 2 3 4 5 6¥ 0 1 2 3 4 50 1 2 3 4 50 1 2 3 4 51 2 3 4 5 6¥ 0 1 2 3 4 50 1 2 3 4 50 1 2 3 4 5 6 Answer: 2.5 Fraction line Decimal point Write answer in boxes. Grid in result. Answer: 201 Either position is correct. 7 12 1 2 3 4 5 6 7 8 90 1 2 3 4 5 6 7 8 90 1 2 3 4 5 6 7 8 90 1 2 3 4 5 6 7 8 91 2 3 4 5 6 7 8 90 1 2 3 4 5 6 7 8 90 1 2 3 4 5 6 7 8 90 1 2 3 4 5 6 7 8 91 2 3 40 1 2 3 40 1 2 3 40 1 2 3 40 1 2 3 40 1 2 30 1 2 3 1 2 3 Answer: • Below are the actual directions that you will find on the test—read them carefully. ● Do your best to be certain of your answer before you grid it. If you erase your answer, do so com- pletely. Incomplete erasures may be picked up by the scoring machines as intended answers. ● Check your work if your answer does not fit on the grid. If you obtain a negative value, a value greater than 9999, or an irrational number, you have made an error. ● Make an educated guess if you don’t know the answer. On student-produced response (grid-in) questions, you don’t lose points for wrong answers. ● Always enter your answer on the grid. Only answers entered on the grid are scored. Your hand- written answer at the top of the grid isn’t scored. However, writing your answer at the top of the grid may help you avoid gridding errors. Approaches to Student-Produced Response Questions ● Decide in which column you want to begin grid- ding your answers before the test starts. This strategy saves time. We recommend that you grid the first (left-hand) column of the grid or that you right-justify your answers. ● If the answer is zero, grid it in column 2, 3, or 4. Zero has been omitted from column 1 to encour- age you to grid the most accurate values for rounded answers. For example, an answer of 1/8 could also be gridded as .125 but not as 0.12, which is less accurate. ● A fraction does not have to be reduced unless it will not fit the grid. For example, 15/25 will not fit. You can grid 3/5, 6/10, or 9/15. The decimal form, .6, can also be gridded.

SAT Preparation Booklet 24 9. For all positive integers a and b, let a  b be defined by a  b =+ − a a b 1 1. What is the value of 4  2? 1 2 3 4 5 6 7 8 90 1 2 3 4 5 6 7 8 90 1 2 3 4 5 6 7 8 90 1 2 3 4 5 6 7 8 9 7 13 / 1 2 3 4 5 6 7 8 90 1 2 3 4 5 6 7 8 90 1 2 3 4 5 6 7 8 90 1 2 3 4 5 6 7 8 9 6 56 1 2 3 4 5 6 7 8 90 1 2 3 4 5 6 7 8 90 1 2 3 4 5 6 7 8 90 1 2 3 4 5 6 7 8 9 7 56 The words “let a  b be defined by ” tell you that the symbol  is not supposed to represent a common mathematical operation but one that is made up for this question. To evaluate 4  2, you substitute 4 for a and 2 for b in the expression a ab+ −1 1. This gives 41 412+ − , which equals 17 3 . The answer may be entered in the grid as 17/3 or as 5.66 or 5.67. Difficulty level: Medium 10. Of the 6 courses offered by the music department at her college, Kay must choose exactly 2 of them. How many different combinations of 2 courses are possible for Kay if there are no restrictions on which 2 courses she can choose? 1 2 3 4 5 6 7 80 1 2 3 4 5 6 7 80 1 2 3 4 5 6 7 80 1 2 3 4 5 6 7 8 9999 5 1 There are 6 courses offered; let us refer to them as 1, 2, 3, 4, 5, and 6. One way to find the number of combinations is to list all possible pairings. They are 1-2, 1-3, 1-4, 1-5, 1-6, 2-3, 2-4, 2-5, 2-6, 3-4, 3-5, 3-6, 4-5, 4-6, and 5-6. There are 15 combinations. Note that 1-2 and 2-1 represent the same combination, so only one is in the list. Sample Questions Below are five examples of student-produced response questions. Following each question, you will find a solution and several ways to enter the correct answer. 475 38 1x x −= −= 8. What value of x satisfies both of the equations above? 1 2 3 4 5 6 7 8 90 1 2 3 4 5 6 7 8 90 1 2 3 4 5 6 7 8 90 1 2 3 4 5 6 7 8 9 12 / 1 2 3 4 5 6 7 8 90 1 2 3 4 5 6 7 8 90 1 2 3 4 5 6 7 8 90 1 2 3 4 5 6 7 8 9 5 Since 475x−= , the value of 47 5 5x−−is either or . 475 412 3 x x x −= = = or 47 5 42 1 2 x x x −=− = = The two values of x that satisfy the first equation are 3 and 1 2. Since the value of is either38 1 38 1−= −xx,or−1. 38 1 82 1 4 −= = = x x x or 38 1 84 1 2 −=− = = x x x The two values of x that satisfy the second equation are 1 4 and 1 2. You are asked to find the value of x that satisfies both equations. That value is 1 2. The answer can be entered in the grid as 1/2 or .5. Difficulty level: Hard

The Mathematics Section 25 You could also notice that there are 5 pairings that start with course 1 and 4 additional pairings that start with course 2, and so forth. The total number of combinations is 5432115++++=. You could also solve the problem by noting that the total number of permutations (that is, the number of differ- ent ways 2 of 6 courses could be selected) is 6 for the first course selected times 5 for the second course selected, or 65 30×=. To find the number of combinations, you must divide the number of permutations by the number of arrangements. For each pair of courses A-B selected, the arrangement B-A is also possible. Therefore, there are 2 arrangements. So the number of combinations is 30 2 15 ÷ =. Difficulty level: Medium 11. Let the function be defined byffxxx () =− 2 7+ + + () =10 10. , If what is one possible vaftl lue oft? 1 2 3 4 5 6 7 8 90 1 2 3 4 5 6 7 8 90 1 2 3 4 5 6 7 8 90 1 2 3 4 5 6 7 8 9 1 1 2 3 4 5 6 7 8 90 1 2 3 4 5 6 7 8 90 1 2 3 4 5 6 7 8 90 1 2 3 4 5 6 7 8 9 4 Since fx x x() =−+ 2 710 , substituting t+() 1 for x into the function yields ft t t+() =+() −+ () + 117110 2 , or ft t t t+() =++() −+() + 1217710 2 , or ft t t+() =−+ 154 2 . Since ft+() = 10, it follows that tt2 540 −+=, or tt−() −() = 140. Therefore, tt==14or . Another way to solve the question would be to use a dummy variable k. For example, let kt=+ 1. fk k k k k() =−+=− () − () 2 710 5 2. Since kt=+1 and ft+() = 10, it follows that fk() =0. So kk−() − () = 520, and therefore, kk==52or . Since ortk t t=− = =14 1,. This question asks for one possible value of t. Either 1 or 4 satisfies the question being asked. Choose only one correct answer (not both) to enter in the grid. When there is a range of possible correct answers, your gridded response must lie within the range. For example, consider a problem for which all numbers between 4 and 5, exclusive, are correct answers. For this problem, although 4.0002 is within the range 45

SAT Preparation Booklet 26 The Writing Section The writing section includes both multiple-choice ques- tions and a direct writing measure in the form of an essay. The multiple-choice sections include: ● Improving sentences (25 questions) ● Identifying sentence errors (18 questions) ● Improving paragraphs (6 questions) The multiple-choice sections measure your ability to ● communicate ideas clearly and effectively. ● improve a piece of writing through revision and editing. ● recognize and identify sentence-level errors. ● understand grammatical elements and structures and how they relate to each other in a sentence. ● recognize correctly formed grammatical structures. ● clearly express ideas through sentence-combining and use of transitional words and phrases. ● improve coherence of ideas within and among paragraphs. Note: Calculators may not be on your desk or be used on the writing section of the SAT. Characteristics of Effective Writing Multiple-choice writing questions focus on common problems associated with four characteristics of effective writing. Illustrations of problems are given below. The fifth category of questions requires recognition of correct sentences and effective writing strategies. Writing problem Sentence illustrating the problem Should be... 1. Being consistent Sequence of tenses After he broke his arm, he is home for two weeks. After he broke his arm, he was home for two weeks. Shift of pronoun If you are tense, one should try to relax. If you are tense, you should try to relax. Parallelism She skis, plays tennis, and flying hang gliders. She skis, plays tennis, and flies hang gliders. Noun agreement Carmen and Sarah are both a pilot. Carmen and Sarah are both pilots. Pronoun reference Several people wanted the job, so he or she filled out the required applications.Several people wanted the job, so they filled out the required applications. Subject-verb agreement There is eight people on the shore. There are eight people on the shore. 2. Expressing ideas logically Coordination and subordination Tawanda has a rash, and she is probably allergic to something.Tawanda has a rash; she is probably allergic to something. Logical comparison Nathan grew more vegetables than his neighbor’s garden.Nathan grew more vegetables than his neighbor grew. Modification and word order Barking loudly, the tree had the dog's leash wrapped around it.Barking loudly, the dog wrapped its leash around the tree. 3. Being clear and precise Ambiguous and vague pronouns In the newspaper they say that few people voted. The newspaper reported that few people voted. Diction He circumvented the globe on his trip. He circumnavigated the globe on his trip. Wordiness There are many problems in the contemporary world in which we live.There are many problems in the contemporary world. Improper modification If your car is parked here while not eating in the restaurant, it will be towed away.If you park here and do not eat in the restaurant, your car will be towed away. 4. Following conventions Pronoun case He sat between you and I at the stadium. He sat between you and me at the stadium. Idiom Natalie had a different opinion for her. Natalie had a different opinion of her. Comparison of modifiers Of the sixteen executives, Naomi makes more money. Of the sixteen executives, Naomi makes the most money. Sentence fragment Fred having to go home early. Fred has to go home early. Comma splice Mary took time out of her busy schedule to visit her aunt, John decided to continue working through the summer.Mary took time out of her busy schedule to visit her aunt, but John decided to continue working through the summer. 5. Recognizing effective writing Some sentences require students to recognize that there is no error.

The Writing Section 27 Improving Sentences This question type measures your ability to ● recognize and correct faults in grammar and sentence structure. ● recognize effective sentences that follow the con- ventions of standard written English. Directions The following sentences test correctness and effective- ness of expression. Part of each sentence or the entire sentence is underlined; beneath each sentence are five ways of phrasing the underlined material. Choice A repeats the original phrasing; the other four choices are different. If you think the original phrasing pro- duces a better sentence than any of the alternatives, select choice A; if not, select one of the other choices. In making your selection, follow the requirements of standard written English; that is, pay attention to grammar, choice of words, sentence construction, and punctuation. Your selection should result in the most effective sentence—clear and precise, without awkward- ness or ambiguity. EXAMPLE: Laura Ingalls Wilder published her first book and she was sixty-five years old then. (A) and she was sixty-five years old then (B) when she was sixty-five (C) at age sixty-five years old (D) upon the reaching of sixty-five years (E) at the time when she was sixty-five a , c d e Answering Improving Sentences Questions Look carefully at the underlined portion of the sentence because it may have to be revised. Keep in mind that the rest of the sentence stays the same. Follow the two steps below in answering each improving sentences question. Step 1: Read the entire sentence carefully but quickly and ask yourself whether the underlined portion is correct or whether it needs to be revised. In the example above, connecting the two ideas (“Laura Ingalls Wilder published her first book”) and (“she was sixty-five years old then”) with the word “and” indicates that the two ideas are equally important. The word “and” should be replaced to establish the relationship between the two ideas. Step 2: Read choices (A) through (E), replacing the under- lined part with each answer choice to determine which revision results in a sentence that is clear and precise and meets the requirements of standard written English. Remember that choice (A) is the same as the underlined portion. Even if you think that the sentence does not require correction and choice (A) is the correct answer, it is a good idea to read each choice quickly to make sure. ● The word “and” indicates that the two ideas it con- nects are equally important. No. ● Replacing the word “and” with “when” clearly expresses the information that the sentence is intended to convey by relating Laura Ingalls Wilder’s age to her achievement. Yes, but continue to look at the other revisions. ● Using the word “at” results in a phrase that is not idiomatic. No. ● The phrase “upon the reaching of ” also results in a phrase that is not idiomatic. No. ● The phrase “at the time when she was sixty-five” is awkward and wordy. No. Correct answer: (B) / Difficulty level: Easy Sample Questions 1. Scenes from the everyday lives of African Americans, which are realistically depicted in the paintings of Henry Ossawa Tanner. (A) Scenes from the everyday lives of African Americans, which are realistically depicted in the paintings of Henry Ossawa Tanner. (B) Scenes from the everyday lives of African Americans being realistically depicted in the paintings of Henry Ossawa Tanner. (C) The paintings of Henry Ossawa Tanner realistically depict scenes from the everyday lives of African Americans. (D) Henry Ossawa Tanner, in his realistic paintings, depicting scenes from the everyday lives of African Americans. (E) Henry Ossawa Tanner, whose paintings realistically depict scenes from the everyday lives of African Americans. For a sentence to be grammatically complete, it must include both a subject and a main verb. When a sentence lacks either a subject or a main verb, the result is a sentence fragment. In this example, all options but (C) are sentence fragments.

SAT Preparation Booklet 28 Directions The following sentences test your ability to recognize grammar and usage errors. Each sentence contains either a single error or no error at all. No sentence contains more than one error. The error, if there is one, is underlined and lettered. If the sentence con- tains an error, select the one underlined part that must be changed to make the sentence correct. If the sentence is correct, select choice E. In choosing answers, follow the requirements of standard written English. EXAMPLE: The other delegates and him immediately A B C accepted the resolution drafted by the D neutral states. No error E a , c d e Answering Identifying Sentence Errors Questions Ask yourself if any of the underlined words and phrases in the sentence contains a grammar or usage error. Follow the two steps below in answering each identifying sentence errors question. Step 1: Read the entire sentence carefully but quickly, pay- ing attention to underlined choices (A) through (D). Keep in mind that some sentences do not contain an error. In the example above, “The other delegates and him” are the people who “immediately accepted the resolution,” and the phrase “drafted by the neutral states” describes “the resolu- tion.” Check each underlined word or phrase for correctness. ● The phrase “The other” correctly modifies the word “delegates.” ● The pronoun “him” is in the wrong case. (One would not say “him immediately accepted.”) “Him” is an error, but go on to check the other choices, especially if you are not sure. ● The word “immediately” correctly modifies the verb “accepted.” ● The phrase “drafted by” correctly expresses the action of the “neutral states.” Step 2: Select the underlined word or phrase that needs to be changed to make the sentence correct. Mark (E) No error if you believe that the sentence is correct as written. In this case, select choice (B) because the underlined word “him” must be changed to “he” to make the sentence correct. Correct answer: (B) / Difficulty level: Easy ● In (A), the phrase “Scenes . . . Americans” is modi- fied by the dependent clause “which . . . Tanner,” but there is no main verb. ● In (B), the phrase “Scenes . . . Tanner” contains no main verb. ● In (D), the noun “Henry Ossawa Tanner” is modified by “depicting” but is not combined with a main verb. ● And in (E), the noun “Henry Ossawa Tanner” is modified by the dependent clause “whose . . . Americans” but not combined with a main verb. ● (C) is correct. It is the only choice in which a sub- ject (“The paintings of Henry Ossawa Tanner”) is combined with a verb (“depict”) to express a com- plete thought. Correct answer: (C) / Difficulty level: Medium 2. Looking up from the base of the mountain, the trail seemed more treacherous than it really was. (A) Looking up (B) While looking up (C) By looking up (D) Viewing (E) Viewed When a modifying phrase begins a sentence, it must logi- cally modify the sentence’s subject; otherwise, it is a dan- gling modifier. In this example, every option except (E) is a dangling modifier. ● In (A), the phrase “Looking up from the base of the mountain” does not logically modify the sub- ject “the trail.” A person might stand at the base of a mountain and look up at a trail, but it is illogical to suggest that a trail looks up from the base of a mountain. ● (B), (C), and (D) are simply variations of the error found in (A). Each results in a sentence that illogi- cally suggests that a trail was looking up from the base of a mountain. ● (E) is correct. Although a trail cannot itself look up from the base of a mountain, a trail can be viewed by someone looking up from the base of a moun- tain, so the phrase “Viewed from the base of the mountain” logically modifies the subject “the trail.” Correct answer: (E) / Difficulty level: Hard Identifying Sentence Errors This question type measures your ability to ● recognize faults in grammar and usage. ● recognize effective sentences that follow the conventions of standard written English.

The Writing Section 29 Sample Questions 3. The students have discovered that they can address A B issues more effectively through letter-writing C campaigns and not through public D demonstrations. No error E ● The error in this sentence occurs at (D). When a comparison is introduced by the adverb “more,” as in “more effectively,” the second part of the com- parison must be introduced by the conjunction “than” rather than “and not.” ● The other options contain no errors. In (A), the plural verb “have discovered” agrees with the plural sub- ject “students.” In (B), the plural pronoun “they” correctly refers to the plural noun “students.” In (C), the preposition “through” appropriately expresses the means by which issues are addressed. ● The sentence may be corrected as follows: The stu- dents have discovered that they can address issues more effectively through letter-writing campaigns than through public demonstrations. Correct answer: (D) / Difficulty level: Medium 4. After hours of futile debate, the committee has A decided to postpone further discussion B of the resolution until their next meeting. C D No error E ● The error in this sentence occurs at (D). A pro- noun must agree in number (singular or plural) with the noun to which it refers. Here, the singular verb “has” establishes “the committee” as a singular noun; therefore, the plural pronoun “their” is used incorrectly. ● The other options contain no errors. In (A), the preposition “After” appropriately introduces a phrase that indicates when the committee made its decision. In (B), “to postpone” is the verb form needed to complete the description of the commit- tee’s decision. In (C), the prepositional phrase “of the resolution” appropriately specifies the subject of the postponed discussion. ● The sentence may be corrected as follows: After hours of futile debate, the committee has decided to postpone further discussion of the resolution until its next meeting. Correct answer: (D) / Difficulty level: Hard Improving Paragraphs This type of question measures your ability to ● edit and revise sentences in the context of a para- graph or entire essay. ● organize and develop paragraphs in a coherent and logical manner. ● apply the conventions of standard written English. Directions The following passage is an early draft of an essay. Some parts of the passage need to be rewritten. Read the passage and select the best answers for the questions that follow. Some questions are about particular sentences or parts of sentences and ask you to improve sentence structure or word choice. Other questions ask you to consider organization and development. In choosing answers, follow the requirements of standard written English. Answering Improving Paragraphs Questions To answer the improving paragraphs questions that accompany the draft essay, you will need to note what sentences need to be corrected and to know how each of the sentences relates to one another and to the essay as a whole. Follow the steps below to answer the questions. Step 1: Read the entire essay quickly to determine its over- all meaning. The essay is intended as a draft, so you will notice errors. Step 2: In answering each question, make sure that your answer about a particular sentence or group of sentences makes sense in the context of the passage as a whole. Choose the best answer from among the choices given, even if you can imagine another correct response. Sample Questions Questions 5-7 are based on the following passage: (1) Many times art history courses focus on the great “masters,” ignoring those women who should have achieved fame. (2) Often women artists like Mary Cassatt have worked in the shadows of their male contemporaries. (3) They have rarely received much attention during their lifetimes. (4) My art teacher has tried to make up for it by teaching us about women artists and their work. (5) Recently she came to class very excited; she had just read about a little- known artist named Annie Johnson, a high school teacher who had lived all of her life in New Haven, Connecticut. (6) Johnson never sold a painting, and her obituary in

SAT Preparation Booklet 30 1937 did not even mention her many paintings. (7) Thanks to Bruce Blanchard, a Connecticut businessman who bought some of her watercolors at an estate sale. (8) Johnson is finally starting to get the attention that she deserved more than one hundred years ago. (9) Blanchard now owns a private collection of hundreds of Johnson’s works— watercolors, charcoal sketches, and pen-and-ink drawings. (10) There are portraits and there are landscapes. (11) The thing that makes her work stand out are the por- traits. (12) My teacher described them as “unsentimental.” (13) They do not idealize characters. (14) Characters are presented almost photographically. (15) Many of the peo- ple in the pictures had an isolated, haunted look. (16) My teacher said that isolation symbolizes Johnson’s life as an artist. 5. In context, which is the best revision to the under- lined portion of sentence 3 (reproduced below)? They have rarely received much attention during their lifetimes. (A) In fact, they had (B) Too bad these artists have (C) As a result, these women have (D) In spite of this, women artists (E) Often it is the case that the former have Although sentence 3 is not grammatically incorrect, its relationship to the preceding sentence needs to be made clearer. A transitional phrase should be added to empha- size the cause-and-effect relationship between the stated facts—women artists received little attention as a conse- quence of having worked in the shadows of their male con- temporaries—and the ambiguous pronoun “They” should be replaced with a word or phrase that clearly refers to the “women artists” and not the “male contemporaries” men- tioned in sentence 2. ● (A), (B), and (D) are unsatisfactory because in each case the transitional phrase (“In fact,” “Too bad,” or “In spite of this”) fails to indicate the cause-and- effect relationship. Moreover, both (A) and (B) leave the ambiguity of the pronoun unresolved. ● (E) is unsatisfactory not only because it fails to signal the cause-and-effect relationship, but also because it is wordy and illogically combines the adverbs “Often” and “rarely.” ● (C) is correct. The transitional phrase “As a result” clearly indicates a cause-and-effect relationship, and “these women” properly resolves the ambiguity of the pronoun “They.” Correct answer: (C) / Difficulty level: Hard 6. In context, which of the following revisions to sen- tence 7 is most needed? (A) Delete “Thanks to”. (B) Move “Thanks to Bruce Blanchard” to the end of sentence 7. (C) Delete “who”. (D) Change “her” to “Johnson’s”. (E) Change the period to a comma and combine sentence 7 with sentence 8. Sentence 7 is a sentence fragment, with neither a subject nor a main verb to finish the thought it has begun. It says “Thanks to Bruce Blanchard,” but it does not say what happened thanks to Bruce Blanchard. It should therefore be joined to an independent clause, complete with sub- ject and verb, that indicates what happened as a result of Blanchard’s action. ● (A), (B), and (D) are unsatisfactory because each fails to provide the main verb needed to complete the sen- tence. Each results in another sentence fragment. ● Although (C) results in a complete sentence, the sentence makes little sense in the context of the paragraph because it suggests that Bruce Blanchard is someone other than the Connecticut business- man who bought the watercolors. ● (E) is correct. This change results in a grammati- cally complete sentence that indicates what hap- pened thanks to Bruce Blanchard’s efforts: Johnson began to get the attention she deserved. Correct answer: (E) / Difficulty level: Medium 7. In context, which of the following is the best version of sentence 10 (reproduced below)? There are portraits and there are landscapes. (A) (As it is now) (B) You can see both portraits and landscapes. (C) Therefore, both portraits and landscapes are among her works. (D) Johnson painted both portraits and landscapes. (E) Among them Johnson has portraits and landscapes. In addition to being vague, sentence 10 contains no noun to which the pronoun “her” in sentence 11 may refer. It should be revised so that Johnson is clearly identified as the painter of the portraits and landscapes. ● (A), (B), and (C) are unsatisfactory because they do not mention Johnson. ● Though (E) does mention Johnson, it is mislead- ing in that the words “Johnson has” suggest that Johnson is the owner rather than the painter of the portraits and landscapes. ● (D) is correct because it properly identifies Johnson as the painter of the artworks and thus provides an antecedent for the pronoun “her” in sentence 11. Correct answer: (D) / Difficulty level: Easy

The Writing Section 31 The Essay The essay measures your ability to ● develop a point of view on an issue presented in an excerpt. ● support your point of view using reasoning and examples from your reading, studies, experience, or observations. ● follow the conventions of standard written English. Approaches to the Essay There are no short cuts to success on the SAT essay. You will not receive high scores on your essay just because it is long, or has five paragraphs, or uses literary examples. The high school and college teachers who score the SAT reward essays that insightfully develop a point of view with appro- priate reasons and examples and that use language skill- fully. So what can you do to write a successful SAT essay? ● Read the entire assignment. It’s all there to help you. Every essay assignment contains a short paragraph about the issue. Imagine that you are talking to the author of the paragraph about the issue. Would you argue with him or her, or agree? What other ideas or examples would you bring up? Answering these questions will help you develop your own point of view. ● Don’t oversimplify. Developing your point of view doesn’t mean coming up with as many examples as you can. Rushing to give multiple relevant exam- ples can lead you to oversimplify a complex topic. An essay with one or two thoughtful, well- developed reasons or examples is more likely to get a high score than an essay with three short, simplistic examples. ● There’s nothing wrong with “I.” You are asked to develop your point of view on the issue, not give a straight report of the facts. This is your opinion, so feel free to use “I,” and give examples that are meaningful to you, even ones from your personal life or experiences. Of course you need to support your ideas appropriately and show that you can use language well, but remember: the essay is an opportunity for you to say what you think about an issue relevant to your life. Directions The essay gives you an opportunity to show how effectively you can develop and express ideas. You should, therefore, take care to develop your point of view, present your ideas logically and clearly, and use language precisely. Your essay must be written on the lines provided on your answer sheet—you will receive no other paper on which to write. You will have enough space if you write on every line, avoid wide margins, and keep your handwriting to a reasonable size. Remember that people who are not familiar with your handwriting will read what you write. Try to write or print so that what you are writing is legible to those readers. Important Reminders: • A pencil is required for the essay. An essay written in ink will receive a score of zero. • Do not write your essay in your test book. You will receive credit only for what you write on your answer sheet. • An off-topic essay will receive a score of zero. • If your essay does not reflect your original and individual work, your test scores may be canceled. You have 25 minutes to write an essay on the topic assigned below. Think carefully about the issue presented in the fol- lowing excerpt and the assignment below. Many persons believe that to move up the ladder of success and achievement, they must forget the past, repress it, and relinquish it. But others have just the opposite view. They see old memories as a chance to reckon with the past and integrate past and present. Adapted from Sara Lawrence-Lightfoot, I’ve Known Rivers: Lives of Loss and Liberation Assignment: Do memories hinder or help people in their effort to learn from the past and succeed in the present? Plan and write an essay in which you develop your point of view on this issue. Support your position with reasoning and examples taken from your reading, studies, experience, or observations. Receive immediate essay scoring for this essay prompt and many more in The Offi cial SAT Online Course ™. Learn more at collegeboard.com/satonlinecourse.

SAT Preparation Booklet 32Sample Essays Score of 6: Without our past, our future would be a tortuous path leading to nowhere. In order to move up the ladder of success and achievement we must come to terms with our past and integrate it into our future. Even if in the past we made mis- takes, this will only make wiser people out of us and guide us to where we are supposed to be. This past year, I was auditioning for the fall play, “Cat on a Hot Tin Roof.” To my detriment I thought it would be a good idea to watch the movie in order to prepare. For two hours I studied Elizabeth Taylor’s mannerisms, attitude, and diction, hoping I could mimic her performance. I audi- tioned for the part of “Maggie” feeling perfectly confident in my portrayal of Elizabeth Taylor, however, I was unaware that my director saw exactly what I had been thinking. Unfortunately, I didn’t get the part, and my director told me that he needed to see “Maggie” from my perspective, not Elizabeth Taylor’s. I learned from this experience, and promised myself I would not try to imitate another actress, in order to create my character. Perservering, I was anxious to audition for the winter play just two months later. The play was Neil Simon’s “Rumors,” and would get the opportunity to play “Chris,” a sarcastic yet witty role, which would be my final perfor- mance in high school. In order to develop my character, I planned out her life just as I thought it should be, gave her the voice I thought was right, and the rest of her char acter unfolded beautifully from there. My director told me after the first show that “Rumors” was the best work he’d ever seen from me, and that he was amazed at how I’d devel- oped such a believable character. Thinking back to my first audition I was grateful for that chance I had to learn and to grow, because without that mistake I might have tried to base “Chris” off of someone I’d known or something I’d seen instead of becoming my own character. I utilized the memo- ry of the Elizabeth Taylor debacle to improve my approach to acting and gave the best performance of my life so far. This essay effectively and insightfully develops its point of view (In order to move up the ladder of success and achieve- ment we must come to terms with our past and integrate it into our future) through a clearly appropriate extended example drawing on the writer’s experience as an actor. The essay exhibits outstanding critical thinking by pre- senting a well-organized and clearly focused narrative that aptly illustrates the value of memory. The essay also uses language skillfully, demonstrating meaningful variety in sentence structure (To my detriment I thought it would be a good idea to watch the movie in order to prepare. For two hours I studied Elizabeth Taylor’s mannerisms, attitude, and diction, hoping I could mimic her performance. I auditioned for the part of “Maggie” feeling perfectly confident in my por- trayal of Elizabeth Taylor, however, I was unaware that my director…). Despite minor errors, the essay demonstrates clear and consistent mastery and is scored a 6. Score of 6: Memories act as both a help and a hinderance to the success of someone. Many people advise you to learn from the past and apply those memories so that you can effectively succeed by avoiding repeating your past mistakes. On the other hand, people who get too caught up with the past are unable to move on to the future. Elie Wiesel’s memoir Night perfectly exemplifies the double nature of memories. Wiesel, a Jewish man, suffered heavily throughout the Holocaust and Night is rife with horrific descriptions of his experience. These memories help to spread the view of what life was like. Through recounting these memories, Wiesel is able to educate world readers about the atrocities committed in hopes that the same blatant violations of human rights are never repeated again. Through reliving the Holocaust through his writing, Wiesel was inspired to become proactive in the battle for civil rights. Some would point to his peaceful actions and the sales of his book and label him a success. Despite the importance of recounting such memories, Wiesel acknowledges the damage that memories can also cause. Following his liberation from the Auschwitz concen- tration camp, Wiesel was a bitter, jaded man. He could not even write Night until several years later. The end of the novel describes Wiesel’s gradual but absolute loss of faith throughout the experience. His past experiences haunted him for several years, rendering him passive. It was not until he set aside his past that he could even focus on the future. Had he remained so consumed with the pain and damage caused in the past, he may never have achieved the success that he has attained. Overall, Wiesel’s experiences exemplify the importance of the past as a guide. Wiesel’s past experiences helped to guide him in later life, but it was not until he pushed them aside that he could move on. To me this means that you should rely on your past without letting it control you. Allow your past to act as a guide, while making sure that you are also living in the present and looking to the future. This essay exhibits outstanding critical thinking by effec- tively and insightfully developing its point of view (you should rely on your past without letting it control you) through the clearly appropriate example of Elie Wiesel’s Holocaust memoir, Night. The essay demonstrates clear coherence and smooth progression of ideas, carefully contrasting Wiesel’s success in using his memories to gain attention for his cause with the difficulty Wiesel faced in dealing with those same powerful memories. The essay uses language skillfully to convey Wiesel’s struggle (Despite the importance of recounting such memories, Wiesel acknowl- edges the damage that memories can also cause. Following his liberation from the Auschwitz concentration camp, Wiesel was a bitter, jaded man. He could not even write Night until several years later). The essay demonstrates clear and con- sistent mastery and receives a 6.

The Writing Section 33 Score of 5: Memories and past experiences serve as a rail, a guid- ing support, for people in an effort to succeed in the present. People not only learn from the past, but the very act of going through something provides experience for a person who is to “move up the ladder of success and achievement”. Some view failed experiences as a hinderance to future success. This is very untrue because history has a tendency of repeating itself, and in recognizing past failures, one can learn how to successfully approach similar situations in the future. An example of this is looking back in history to WWI. Sedition acts at this time allowed for the imprison- ment of anyone who voiced an opinion against the presi- dent, or against the war. America recognized this shady time in its past, and instead of covering it up in a move- ment towards a more democratic nation, these acts were published in textbooks and taught to students. Americans saw the poor judgement of this situation and later with the war in Iraq, approached “patriotism” differently. With this present war, those adverse to the war are able to voice their opinions without fear of imprisonment or death. In seeing the undemocratic ways of an earlier era, America was able to recognize the bad and try to reform it. If the Sedition Acts had been forgotten then what is to say that they wouldn’t come back? Remembering the failed times insures that improvement is possible. In my personal experience, I have found that the very act of living through something not only matures me, but also provides skills and knowledge. In remembering past events, I am able to use them as reference, and sometimes assurance. A personal example, somewhat juvenile, but also effective, is when my first pet died. I was devastated and wanted to just clear my mind of the event, but I didn’t. After time, I recov- ered, but maintained the memory of this horrible tragedy. Later in life, another pet died. I looked back to that memory as a guide and learned from it that in time I would be fine and to just hang on. In this situation, a memory served as a reference and catalyzed in my personal growth and recovery. Memories, good or bad, assist people in obtaining suc- cess. Whether used as reference for guidance, or lessons on what not to do, past experiences can only offer a gap between the steps on the ladder of success. Forgetting the past can and will only erase experience and knowledge from a person and in affect hinder one in seeking achievement. In looking at historical repeats and personal events, it is clear that old memories can only aid in success. This essay effectively develops its point of view (Memories and past experiences serve as a rail, a guiding support, for people in an effort to succeed in the present) through the appropriate examples of dissent during wartime and grieving for a pet, thus demonstrating strong critical think- ing. Well organized and focused, the essay demonstrates coherence and progression of ideas (In seeing the undemo- cratic ways of an earlier era, America was able to recognize the bad and try to reform it. If the Sedition Acts had been forgotten then what is to say that they wouldn’t come back? Remem bering the failed times insures that improvement is possible). The essay also uses appropriate vocabulary and demonstrates effective variety in sentence structure. To earn a 6, this writer needs to achieve smoother progression of ideas by using language more skillfully (the phrase “past experiences can only offer a gap between the steps on the ladder of success” seems to express the opposite of what the writer intends). The essay demonstrates reasonably consistent mastery and receives a 5. Score of 5: I agree with Ms. Sara Lawrence-Lightfoot in saying that some people “see old memories as a chance to reckon with the past and integrate past and present.” Many people are so troubled by things that happened in their past that they are not able to focus on the present. For example, in the book Ceremony, by Leslie Marmon Silko, Tayo, the main character, can not concentrate on the present because he constantly hounds himself over things that happened dur- ing World War II and his troubled childhood. However, past memories can help people to succeed in the present. An historical example of people learning from the past would be the Marshall Plan. After the conclusion of World War II there were many countries around the world in need of eco- nomical assistence to help rebuild their war torn countries, and the United States would have to be the one to provide that assistence. Many American politicians thought it was foolish for the US government to spend money abroad on countries that would not be able to repay the loan for a long time. However, George Marshall, a former general and later Secretary of State under President Truman, remembered how the exact same argument of “why should we spend money on war torn nations that really owe us reparations?” had been used after World War I towards Germany. The lack of assistence towards Germany after World War I had caused a gigantic economic depression in Germany that had made the Mark (German money) virtually worthless. The German people became so desperate that they started supporting an extreme German nationalist named Adolf Hitler, who even- tually started World War II. Marshall knew that if the US did not help war torn Germany and, especially, Japan, we could eventually have a World War III on our hands. This focused essay effectively develops its point of view and demonstrates strong critical thinking (Many people are so troubled by things that happened in their past that they are not able to focus on the present. . . . However, past memories can help people to succeed in the present). The essay uses appropriate reasoning and examples and demonstrates coherence and progression of ideas (Many American politi- cians thought it was foolish for the US government to spend money abroad on countries that would not be able to repay the loan for a long time. However, George Marshall . . . remembered how the exact same argument . . . had been used after World War I towards Germany). The essay also exhibits facility in the use of language. To earn a score of 6, the writer needs to achieve clearer coherence and smoother progression of ideas by integrating the example of Ceremony more effectively into the overall essay, perhaps through an extended comparison of Tayo’s and Marshall’s experiences of World War II. The essay demonstrates rea- sonably consistent mastery and is scored a 5.

SAT Preparation Booklet 34 Score of 4: Interestingly enough, I fall in the middle of these state- ments. I believe that one should remember the past and learn from those events. However, I also believe that many bad memories harm the present and the future. The only way to continue, many times, is to forget and forgive. My brother, who is college, has proved to me the impor- tance of getting good grades and actively participating in extracorrecular activities. These two ideas helped him to get into the prestegious college of the University of Notre Dame. His education there will allow him to have a prosperous career as an adult. Reviewing these facts and ideas has led me to believe if I do the same, I will have a similar promising career. Consequently, I have gotten good grades and have seen interest from many prestigious programs. Through my knowledge, I have learned that in many bad instances, time to forget is very important. Ireland, for example, had been persecuted for many hundreds of years from 1000 AD to 1900 AD. After being granted the Irish Free State, they attacked many parts of Britain for retribution of those many years of being oppressed. Consequently there has been on going hostility between the two peoples. This hostil- ity has cost the lives of many hundreds of people. A quote once said, “Violence begets violence” is the perfect phrase for this warfare. The only way to stop the loss of life is to forget and forgive; start anew. Different situations require different actions to proceed in a positive manner. Many times, people are required to use both elements. For example, let’s forget this part and concentrate on how to bring this positive part into light. Both of the ideas on remembering and forgetting have their reasons for existing and both are positive. This essay provides adequate reasons and examples to support both aspects of its point of view (I believe that one should remember the past and learn from those events. However, I also believe that many bad memories harm the present and the future), thus demonstrating competent crit- ical thinking. The essay is generally organized and focused and features coherence and progression of ideas. Facility in the use of language is adequate, despite some inconsisten- cies (Through my knowledge, I have learned that in many bad instances, time to forget is very important). The essay also has some errors in grammar, usage, and mechanics. To earn a higher score, the writer should provide additional appropriate evidence and use critical thinking to extend the discussion of situations in which “people are required to use both elements.” The essay demonstrates adequate mastery and receives a 4. Score of 4: The point of making mistakes is to learn from them. If you don’t learn from what you do wrong, then making mistakes has no silver lining, it is purely bad. I have come to believe this through personal experience and watching others. When climbing the “ladder of success,” each step gets you closer to the top. Therefore each step is a mistake that you learned from, a good decision, or even a stroke of luck. How could a person climb that ladder without each and every wooden rung to help them? I am human, therefor, far from perfect, I make mistakes all of the time and I am a better person because of that. You could almost say that the more mistakes a person makes, the stronger a person they are, assuming of course that they learn from them. As a child I stole cookies from the cookie jar, lied to my parents (still happens every once in awhile), and played tricks on my brothers. I, in turn, got in trouble with my par- ents and was punished. After that I learned that those things aren’t okay. Now I tend to make different mistakes, such as, going to places that aren’t safe for me, and giving up when things get hard. Life is a huge cycle of making mistakes and learning from them. That is why people can become so wise and strong in what they do, they make good out of the bad. I also see people close to me using problems and mis- takes to make a good situation out of a bad one. My parents, my brothers, and my closest friends are all slowly building up the knowledge to be successful. How can a person be more successful by forgetting what they have already learned? That doesn’t push you forward it just holds a person back. Even if a person wanted to forget their past, they couldn’t. It’s like forgetting that if a stove is turned on and you touch it, it will burn you. This essay develops a point of view (Life is a huge cycle of making mistakes and learning from them) with adequate reasons and examples, thus demonstrating competent criti- cal thinking. Generally organized and focused around the notion that remembering past learning experiences is cru- cial for success, the essay is marked by coherence and pro- gression of ideas (As a child I stole cookies from the cookie jar, lied to my parents . . . , and played tricks on my brothers. I, in turn, got in trouble with my parents and was punished. After that I learned that those things aren’t okay. Now I tend to make different mistakes). The essay also exhibits adequate facility in the use of language, despite some errors (I am human, therefor, far from perfect, I make mistakes all of the time and I am a better person because of that). To attain a higher score, the writer needs to support and extend the essay’s argument with additional focused examples of peo- ple learning, or not learning, from their experiences. The essay demonstrates adequate mastery and is scored a 4.

The Writing Section 35 Score of 3: Memories can be helpful to some and hinder others. I believe that memories from different aspects of ones life have different consequences. One memory may be bad and it may be best forgotten about, when trying to succeed. Though some memories may give on strength to suceed in achieving a higher status in life. When a person completes a task they have done once before, it trigers a memory and lets the reader reflect on that particular time in life. For example, a sporting team at the local high school makes it to the state championships, but severly loses to their opponent, the next time they get to the state championships they may think about the past and how they lost before, and it may hinder there feelings and they may once again lose. This demonstrates how a memory can ruin a certain activity for ever. On the other hand a memory can also help someone to move up the ladder of success. As an example if a person has cancer and is given treatment then diagnosed in remission they feel like they have beat the cancer. When the patient in remission is later told that the cancer has grown back, the patient might feel that they can kill the cancer again because when looking at the past they see they have beat it once why not beat it again. This demon- strates how a memory can be helpful to a person. In this case it did not help the person climb the ladder of success though it helped the to continue climbing the ladder of life to the extent that they were able to climb. Those two short examples just go to demonstrate how memories of the past can both help and hinder a person in their path of not only success but also in the path of life. This essay develops a point of view (Memories can be help- ful to some and hinder others) and shows some critical thinking by providing examples of the positive and nega- tive effects of memories. However, the examples are limited in focus, featuring some lapses in coherence and progres- sion of ideas, and are thus inadequate to support the posi- tion. The essay also demonstrates occasional problems in sentence structure and mechanics. To achieve a higher score, this writer needs to use critical thinking to clarify and expand each example by adding additional focused reasoning and details. The writer also needs to avoid using run-on sentences (. . . when looking at the past they see they have beat it once why not beat it again). The essay demon- strates developing mastery and earns a 3. Score of 2: I think it is wrong to believe that to move up the ladder of success and achievement, that they must forget the past, repress it, and relinquish it. Everything you did and saw in the past helps you to move on. Every single happy moment, every mistake you make is getting a part of you. Your actions become habits which creates your personality and helps you to make your own experience. Therefore memories help peo- ple in their effort to learn from the past and succeed in the present. Everything we do has to do with our experiences in the past, the way we get along with people or treat them, the way we turn out to be an adult. If you don’t live with mak- ing your own decisions, mistakes, and your experience with people and the world or school you won’t have any examples to compare or to handle any coming situations in the future. If you get everything told you by someone, you will always wait for other people to make decisions for you and won’t have your own point of view. For succeed you have to know what you want, to find that out, you have to have been through some difficult situations in the PAST. Although it expresses a point of view (I think it is wrong to believe that to move up the ladder of success and achieve- ment, that they must forget the past, repress it, and relinquish it), this essay is seriously limited, exhibiting weak critical thinking, insufficient use of evidence, and serious problems with progression of ideas. The essay also demonstrates fre- quent problems in usage, grammar, and sentence structure. To achieve a higher score, the writer needs to develop the point of view with reasons and specific examples instead of merely repeating the same vague ideas (Everything you did and saw in the past helps you to move on. . . . Everything we do has to do with our experiences in the past). The essay demonstrates little mastery and is scored a 2. Score of 1: My oppion on this topic are oposing memories and favoring them. People do succed with repeating their memo- ries. They might have horrible memories but also succeed because they don’t repeat the past. I also think memories should not rule the present. If you let the past overcome the preset you won’t get any where. This is why memories should be guidelines, not rules. If you repeat the past it won’t come out as well as it did because the world has changed. See the past will never change with the world, but the world will change to overcome the past. So in conclusion don’t forget the past or live in it, and the past is only guidelines. This minimal essay demonstrates very little mastery, offer- ing only a collection of general ideas in support of the writer’s point of view (don’t forget the past or live in it, and the past is only guidelines). The evidence presented is disor- ganized and unfocused, resulting in a disjointed essay. To earn a higher score, this writer needs to provide additional focused evidence that develops the point of view, including specific examples. The essay demonstrates very little mas- tery and receives a 1.

SAT Preparation Booklet 36 Scoring the Essay Essays are scored in a manner that is fair and consistent, using a holistic approach. In holistic scoring, a piece of writing is considered as a total work, the whole of which is greater than the sum of its parts. Essays are scored by expe- rienced high school teachers and college faculty members. The majority of essay readers teach English, composition, or language arts courses. Each essay is scored indepen dently by two readers on a scale of 1 to 6, with 6 being the highest score. The combined score for both readers will range from 2 to 12. If the two readers’ scores are more than one point apart, a third reader resolves the discrepancy. In scoring the essays, readers follow the scoring guide below. The scoring guide describes the features typically found in essays at each score point, including critical thinking, devel- opment, organization, language use, and sentence structure. A student can get a top score on the essay even with minor errors in grammar, usage, and mechanics. The SAT essay neither rewards nor penalizes formulaic approaches to writ- ing, such as the five-paragraph essay. There is no formula for effective writing, no single best way to communicate an idea. Any essay that features clear lines of reasoning, appropriate choices of evidence, ample development of ideas, effective organization, and precise use of language will receive a high score, regardless of style or approach. Readers are trained to recognize and reward a wide variety of essays at each score point. SCORING GUIDE SCORE OF 6 SCORE OF 5 SCORE OF 4 An essay in this category demonstrates clear and consistent mastery, although it may have a few minor errors. A typical essayAn essay in this category demonstrates reason- ably consistent mastery, although it will have occasional errors or lapses in quality. A typical essayAn essay in this category demonstrates adequate mastery, although it will have lapses in quality. A typical essay effectively and insightfully develops a point of view on the issue and demonstrates outstanding critical thinking, using clearly appropriate examples, reasons, and other evidence to support its position effectively develops a point of view on the issue and demonstrates strong critical thinking, generally using appropriate examples, reasons, and other evidence to support its position develops a point of view on the issue and demonstrates competent critical thinking, using adequate examples, reasons, and other evidence to support its position is well organized and clearly focused, demonstrating clear coherence and smooth progression of ideas is well organized and focused, demonstrating coherence and progression of ideas is generally organized and focused, demon- strating some coherence and progression of ideas exhibits skillful use of language, using a varied, accurate, and apt vocabulary exhibits facility in the use of language, using appropriate vocabulary exhibits adequate but inconsistent facility in the use of language, using generally appropri- ate vocabulary demonstrates meaningful variety in sentence structure demonstrates variety in sentence structure demonstrates some variety in sentence structure is free of most errors in grammar, usage, and mechanics is generally free of most errors in grammar, usage, and mechanics has some errors in grammar, usage, and mechanics SCORE OF 3 SCORE OF 2 SCORE OF 1 An essay in this category demonstrates developing mastery, and is marked by ONE OR MORE of the following weaknesses:An essay in this category demonstrates little mastery, and is flawed by ONE OR MORE of the following weaknesses:An essay in this category demonstrates very little or no mastery, and is severely flawed by ONE OR MORE of the following weaknesses: develops a point of view on the issue, demon- strating some critical thinking, but may do so inconsistently or use inadequate examples, reasons, or other evidence to support its position develops a point of view on the issue that is vague or seriously limited, and demonstrates weak critical thinking, providing inappropriate or insufficient examples, reasons, or other evidence to support its position develops no viable point of view on the issue, or provides little or no evidence to support its position is limited in its organization or focus, or may demonstrate some lapses in coherence or progression of ideas is poorly organized and/or focused, or demonstrates serious problems with coherence or progression of ideas is disorganized or unfocused, resulting in a disjointed or incoherent essay displays developing facility in the use of lan- guage, but sometimes uses weak vocabulary or inappropriate word choice displays very little facility in the use of language, using very limited vocabulary or incorrect word choice displays fundamental errors in vocabulary lacks variety or demonstrates problems in sentence structure demonstrates frequent problems in sentence structure demonstrates severe flaws in sentence structure contains an accumulation of errors in grammar, usage, and mechanics contains errors in grammar, usage, and mechanics so serious that meaning is somewhat obscured contains pervasive errors in grammar, usage, or mechanics that persistently interfere with meaning Essays not written on the essay assignment will receive a score of zero.

37 Official SAT Practice Test About the Practice Test Take the practice test, which starts on page 46, to reinforce your test-taking skills and to be more comfortable when you take the SAT. This practice test will give you a good idea of what to expect on the actual test. However, the test you eventually take will differ in some ways. It may, for example, contain a different number of reading passages, and its sections may be in a different order. Also, this practice SAT includes only nine of the ten sections that the actual test contains. Section 7 is an unscored sec- tion and has been omitted on this test because it contains questions that may be used in future editions of the SAT. The practice test will help you most if you take it under conditions as close as possible to those of the actual test. Approaches to the Practice Test ● Set aside 3 hours and 20 minutes of uninterrupted time. That way you can complete the entire test in one sitting. Note: the total testing time is 3 hours and 45 minutes, but you save 25 minutes because the unscored section from this practice test was omitted. ● Sit at a desk or table cleared of any other papers or books. You won’t be able to take a dictionary, books, notes, or scratch paper into the test room. ● Allow yourself the specified amount of time for each section. Pace yourself by using a watch (without an audible alarm), which is what you are allowed on test day. ● Have a calculator at hand when you take the math- ematics sections. This will help you determine how much to use a calculator the day of the test. Use a calculator with which you are familiar. ● Read the test instructions carefully. They are reprinted from the back cover of the test book. On test day, you will be asked to read them before you begin answering questions. ● Make sure you use a No. 2 pencil. It is very impor- tant that you fill in the entire circle on the answer sheet darkly and completely. If you change your response, erase it as completely as possible. It is very important that you follow these instructions when filling out your answer sheet. ● After you finish the test, read page 85 for instructions on how to find your score. If you have access to the Internet, visit collegeboard.com/satpracticetest to review answer explanations or to see sample essays. Finding Your Scores To score your test, you can either enter your answers online at collegeboard.com/satpracticetest and have your test scored automatically, or you can score it yourself with the instructions on page 85. To score the test yourself, you’ll need to count the right and wrong answers for each sec- tion, and then convert your “raw” score to the College Board scale of 200 to 800. With either scoring method, you’ll need to choose a score for your essay. Use the Scoring Guide on page 36 to deter- mine how your particular essay might be scored. Reviewing Your Performance After you score your practice test, review your performance to see where your strengths and weaknesses are. Ask yourself these questions: ● Did you run out of time before you finished a sec- tion? Try to pace yourself so you will have time to answer all the questions you can. Don’t spend too much time on any one question. ● Did you hurry and make careless mistakes? You may have misread the question, neglected to notice a word like “except” or “best,” or solved for the wrong value. ● Did you spend too much time reading directions? You should be familiar with the test directions so you don’t have to spend as much time reading them when you take the actual test. Visit collegeboard.com/satpracticetest to view answer explanations for questions you answered incorrectly and to read sample scored essays. Official SAT Practice Test The Offi cial SAT Online Course ➢ Take this practice test online ➢ Receive an immediate essay score ➢ Practice with more tests and quizzes Visit: collegeboard.com/satonlinecourse

SAT Preparation Booklet 38 FORM CODE TEST CENTER PLEASE DO NOT WRITE IN THIS AREA SERIAL # 2007-08 SAT Reasoning Test TM Female Male SEX 5 REGISTRATION NUMBER 6 (Copy from Admission Ticket.) TEST BOOK SERIAL NUMBER 10 (Copy from front of test book.) 11 (Supplied by Test Center Supervisor.) SOCIAL SECURITY NUMBER 7 FOR OFFICIAL USE ONLY 0 1 2 3 4 5 6 Last Name (First 6 Letters) YOUR NAME 2 A B C D E F G H I J K L M N O P Q R S T U V W X Y Z A B C D E F G H I J K L M N O P Q R S T U V W X Y Z A B C D E F G H I J K L M N O P Q R S T U V W X Y Z A B C D E F G H I J K L M N O P Q R S T U V W X Y Z A B C D E F G H I J K L M N O P Q R S T U V W X Y Z A B C D E F G H I J K L M N O P Q R S T U V W X Y Z A B C D E F G H I J K L M N O P Q R S T U V W X Y Z A B C D E F G H I J K L M N O P Q R S T U V W X Y Z A B C D E F G H I J K L M N O P Q R S T U V W X Y Z A B C D E F G H I J K L M N O P Q R S T U V W X Y Z A B C D E F G H I J K L M N O P Q R S T U V W X Y Z First Name (First 4 Letters)Mid. Init. – ’ – ’ – ’ – ’ – ’ 174267-001:654321 ISD6542 © 2007 The College Board. All rights reserved. College Board, SAT, and the acorn logo are registered trademarks of the College Board. SAT Reasoning Test is a trademark owned by the College Board. I turned in my registration form today. A C H I J K O L M P B F G Q S V E D R N U T W Z Y X A C H I J K O L M P B F G Q S V E D R N U T W Z Y X A C H I J K O L M P B F G Q S V E D R N U T W Z Y X A C H I J K O L M P B F G Q S V E D R N U T W Z Y X 8 ZIP CODE 4 Important: Fill in items 8 and 9 exactly as shown on the back of test book. 00272-36390 • NS67E3600 • Printed in U.S.A. 737667 TEST FORM (Copy from back of test book.) 9 (Copy and grid as on back of test book.) 1 2 3 4 5 6 7 8 9 0 1 2 3 4 5 6 7 8 9 0 1 2 3 4 5 6 7 8 9 0 1 2 3 4 5 6 7 8 9 0 1 2 3 4 5 6 7 8 9 0 1 2 3 4 5 6 7 8 9 0 1 2 3 4 5 6 7 8 9 0 1 2 3 4 5 6 7 8 9 0 1 2 3 4 5 6 7 8 9 0 1 2 3 4 5 6 7 8 9 0 1 2 3 4 5 6 7 8 9 0 1 2 3 4 5 6 7 8 9 0 1 2 3 4 5 6 7 8 9 0 1 2 3 4 5 6 7 8 9 0 1 2 3 4 5 6 7 8 9 0 1 2 3 4 5 6 7 8 9 0 1 2 3 4 5 6 7 8 9 0 1 2 3 4 5 6 7 8 9 0 1 2 3 4 5 6 7 8 9 0 1 2 3 4 5 6 7 8 9 0 1 2 3 4 5 6 7 8 9 0 1 2 3 4 5 6 7 8 9 0 1 2 3 4 5 6 7 8 9 0 1 2 3 4 5 6 7 8 9 0 1 2 3 4 5 6 7 8 9 0 1 2 3 4 5 6 7 8 9 0 1 2 3 4 5 6 7 8 9 0 1 2 3 4 5 6 7 8 9 0 1 2 3 4 5 6 7 8 9 0 1 2 3 4 5 6 7 8 9 0 1 2 3 4 5 6 7 8 9 0 1 2 3 4 5 6 7 8 9 0 1 2 3 4 5 6 7 8 9 0 1 2 3 4 5 6 7 8 9 0 1 2 3 4 5 6 7 8 9 0 1 2 3 4 5 6 7 8 9 0 1 2 3 4 5 6 7 8 9 0 1 2 3 4 5 6 7 8 9 0 0 1 2 3 4 5 6 0 1 2 3 4 5 6 MARKS MUST BE COMPLETE You must use a No. 2 pencil.Do not use a mechanical pencil.It is very important that you fill in the entire circle darkly and completely. If you change your response, erase as completely as possible. Incomplete marks or erasures may affect your score. It is very important that you follow these instructions when filling out your answer sheet. COMPLETE MARKEXAMPLES OF INCOMPLETE MARKS (Print) 1 LastFirst M.I. Your Name: I agree to the conditions on the front and back of the SAT Reasoning Test™ booklet. I also agree to use only a No. 2 pencil to complete my answer sheet. Signature: Home Address:Date: Center:(Print)City State City Number and Street Home Phone: ( ) State/CountryZip Code (Print) (Print) DATE OF BIRTH MONTH DAY YEAR 1 2 3 0 1 2 3 4 5 6 7 8 9 0 3 Jan Feb Mar Apr May Jun Jul Aug Sep Oct Nov Dec5 6 7 8 91 2 3 4 5 6 7 8 9 0 MM DD YY A B C D A B C D CCCCCCCC AA

Official SAT Practice Test 39 Page 2 I prefer NOT to grant the College Board the right to use, reproduce, or publish my essay for any purpose beyond the assessment of my writing skills, even though my name will not be used in any way in conjunction with my essay. I understand that I am free to mark this circle with no effect on my score. Begin your essay on this page. If you need more space, continue on the next page. Continue on the next page, if necessary. SECTION 1 IMPORTANT: USE A NO. 2 PENCIL. DO NOT WRITE OUTSIDE THE BORDER! Words written outside the essay box or written in ink WILL NOT APPEAR in the copy sent to be scored, and your score will be affected.

SAT Preparation Booklet 40 PLEASE DO NOT WRITE IN THIS AREA SERIAL # Page 3 Continuation of ESSAY Section 1 from previous page. Write below only if you need more space. IMPORTANT: DO NOT START on this page—if you do, your essay may appear blank and your score may be affected.

Official SAT Practice Test 41 9 31 32 33 34 35 36 37 38 39 40 21 22 23 24 25 26 27 28 29 30 11 12 13 14 15 16 17 18 19 20 1 2 3 4 5 6 7 8 9 10 1410 1511 1612 1713 18 SECTION 3 Grid answers in the section below for SECTION 2 or SECTION 3 only if directed to do so in your test book. CAUTION Student-Produced Responses ONLY ANSWERS THAT ARE GRIDDED WILL BE SCORED. YOU WILL NOT RECEIVE CREDIT FOR ANYTHING WRITTEN IN THE BOXES. Page 4 31 32 33 34 35 36 37 38 39 40 21 22 23 24 25 26 27 28 29 30 11 12 13 14 15 16 17 18 19 20 1 2 3 4 5 6 7 8 9 10 A B C D E A B C D E A B C D E A B C D E A B C D E A B C D E A B C D E A B C D E A B C D E A B C D E SECTION 2 A B C D E A B C D E A B C D E A B C D E A B C D E A B C D E A B C D E A B C D E A B C D E A B C D E A B C D E A B C D E A B C D E A B C D E A B C D E A B C D E A B C D E A B C D E A B C D E A B C D E A B C D E A B C D E A B C D E A B C D E A B C D E A B C D E A B C D E A B C D E A B C D E A B C D E A B C D E A B C D E A B C D E A B C D E A B C D E A B C D E A B C D E A B C D E A B C D E A B C D E A B C D E A B C D E A B C D E A B C D E A B C D E A B C D E A B C D E A B C D E A B C D E A B C D E A B C D E A B C D E A B C D E A B C D E A B C D E A B C D E A B C D E A B C D E A B C D E A B C D E A B C D E A B C D E A B C D E A B C D E A B C D E A B C D E A B C D E A B C D E A B C D E A B C D E Quality Assurance Mark You must use a No. 2 pencil and marks must be complete.Do not use a mechanical pencil.It is very important that you fill in the entire circle darkly and completely. If you change your response, erase as completely as possible. Incomplete marks or erasures may affect your score. COMPLETE MARKEXAMPLES OF INCOMPLETE MARKS 1 2 3 4 5 6 7 8 9 1 2 3 4 5 6 7 8 9 1 2 3 4 5 6 7 8 9 1 2 3 4 5 6 7 8 9 ⁄ 0 . ⁄ . . 0 0 . 1 2 3 4 5 6 7 8 9 1 2 3 4 5 6 7 8 9 1 2 3 4 5 6 7 8 9 1 2 3 4 5 6 7 8 9 ⁄ 0 . ⁄ . . 0 0 . 1 2 3 4 5 6 7 8 9 1 2 3 4 5 6 7 8 9 1 2 3 4 5 6 7 8 9 1 2 3 4 5 6 7 8 9 ⁄ 0 . ⁄ . . 0 0 . 1 2 3 4 5 6 7 8 9 1 2 3 4 5 6 7 8 9 1 2 3 4 5 6 7 8 9 1 2 3 4 5 6 7 8 9 ⁄ 0 . ⁄ . . 0 0 . 1 2 3 4 5 6 7 8 9 1 2 3 4 5 6 7 8 9 1 2 3 4 5 6 7 8 9 1 2 3 4 5 6 7 8 9 ⁄ 0 . ⁄ . . 0 0 . 1 2 3 4 5 6 7 8 9 1 2 3 4 5 6 7 8 9 1 2 3 4 5 6 7 8 9 1 2 3 4 5 6 7 8 9 ⁄ 0 . ⁄ . . 0 0 . 1 2 3 4 5 6 7 8 9 1 2 3 4 5 6 7 8 9 1 2 3 4 5 6 7 8 9 1 2 3 4 5 6 7 8 9 ⁄ 0 . ⁄ . . 0 0 . 1 2 3 4 5 6 7 8 9 1 2 3 4 5 6 7 8 9 1 2 3 4 5 6 7 8 9 1 2 3 4 5 6 7 8 9 ⁄ 0 . ⁄ . . 0 0 . 1 2 3 4 5 6 7 8 9 1 2 3 4 5 6 7 8 9 1 2 3 4 5 6 7 8 9 1 2 3 4 5 6 7 8 9 ⁄ 0 . ⁄ . . 0 0 . 1 2 3 4 5 6 7 8 9 1 2 3 4 5 6 7 8 9 1 2 3 4 5 6 7 8 9 1 2 3 4 5 6 7 8 9 ⁄ 0 . ⁄ . . 0 0 . A B C D A B C D CCCCCCCC AA

SAT Preparation Booklet 42 Quality Assurance Mark 9 31 32 33 34 35 36 37 38 39 40 21 22 23 24 25 26 27 28 29 30 11 12 13 14 15 16 17 18 19 20 1 2 3 4 5 6 7 8 9 10 1410 1511 1612 1713 18 Grid answers in the section below for SECTION 4 or SECTION 5 only if directed to do so in your test book. Student-Produced Responses Page 5 SECTION 5 ONLY ANSWERS THAT ARE GRIDDED WILL BE SCORED. YOU WILL NOT RECEIVE CREDIT FOR ANYTHING WRITTEN IN THE BOXES. 31 32 33 34 35 36 37 38 39 40 21 22 23 24 25 26 27 28 29 30 11 12 13 14 15 16 17 18 19 20 1 2 3 4 5 6 7 8 9 10 SECTION 4 A B C D E A B C D E A B C D E A B C D E A B C D E A B C D E A B C D E A B C D E A B C D E A B C D E A B C D E A B C D E A B C D E A B C D E A B C D E A B C D E A B C D E A B C D E A B C D E A B C D E A B C D E A B C D E A B C D E A B C D E A B C D E A B C D E A B C D E A B C D E A B C D E A B C D E A B C D E A B C D E A B C D E A B C D E A B C D E A B C D E A B C D E A B C D E A B C D E A B C D E A B C D E A B C D E A B C D E A B C D E A B C D E A B C D E A B C D E A B C D E A B C D E A B C D E A B C D E A B C D E A B C D E A B C D E A B C D E A B C D E A B C D E A B C D E A B C D E A B C D E A B C D E A B C D E A B C D E A B C D E A B C D E A B C D E A B C D E A B C D E A B C D E A B C D E A B C D E A B C D E A B C D E A B C D E A B C D E A B C D E A B C D E A B C D E A B C D E A B C D E You must use a No. 2 pencil and marks must be complete.Do not use a mechanical pencil.It is very important that you fill in the entire circle darkly and completely. If you change your response, erase as completely as possible. Incomplete marks or erasures may affect your score. COMPLETE MARKEXAMPLES OF INCOMPLETE MARKS 1 2 3 4 5 6 7 8 9 1 2 3 4 5 6 7 8 9 1 2 3 4 5 6 7 8 9 1 2 3 4 5 6 7 8 9 ⁄ 0 . ⁄ . . 0 0 . 1 2 3 4 5 6 7 8 9 1 2 3 4 5 6 7 8 9 1 2 3 4 5 6 7 8 9 1 2 3 4 5 6 7 8 9 ⁄ 0 . ⁄ . . 0 0 . 1 2 3 4 5 6 7 8 9 1 2 3 4 5 6 7 8 9 1 2 3 4 5 6 7 8 9 1 2 3 4 5 6 7 8 9 ⁄ 0 . ⁄ . . 0 0 . 1 2 3 4 5 6 7 8 9 1 2 3 4 5 6 7 8 9 1 2 3 4 5 6 7 8 9 1 2 3 4 5 6 7 8 9 ⁄ 0 . ⁄ . . 0 0 . 1 2 3 4 5 6 7 8 9 1 2 3 4 5 6 7 8 9 1 2 3 4 5 6 7 8 9 1 2 3 4 5 6 7 8 9 ⁄ 0 . ⁄ . . 0 0 . 1 2 3 4 5 6 7 8 9 1 2 3 4 5 6 7 8 9 1 2 3 4 5 6 7 8 9 1 2 3 4 5 6 7 8 9 ⁄ 0 . ⁄ . . 0 0 . 1 2 3 4 5 6 7 8 9 1 2 3 4 5 6 7 8 9 1 2 3 4 5 6 7 8 9 1 2 3 4 5 6 7 8 9 ⁄ 0 . ⁄ . . 0 0 . 1 2 3 4 5 6 7 8 9 1 2 3 4 5 6 7 8 9 1 2 3 4 5 6 7 8 9 1 2 3 4 5 6 7 8 9 ⁄ 0 . ⁄ . . 0 0 . 1 2 3 4 5 6 7 8 9 1 2 3 4 5 6 7 8 9 1 2 3 4 5 6 7 8 9 1 2 3 4 5 6 7 8 9 ⁄ 0 . ⁄ . . 0 0 . 1 2 3 4 5 6 7 8 9 1 2 3 4 5 6 7 8 9 1 2 3 4 5 6 7 8 9 1 2 3 4 5 6 7 8 9 ⁄ 0 . ⁄ . . 0 0 . CAUTION A B C D A B C D CCCCCCCC AA

Official SAT Practice Test 43 9 31 32 33 34 35 36 37 38 39 40 21 22 23 24 25 26 27 28 29 30 11 12 13 14 15 16 17 18 19 20 1 2 3 4 5 6 7 8 9 10 1410 1511 1612 1713 18 SECTION 7 Grid answers in the section below for SECTION 6 or SECTION 7 only if directed to do so in your test book. Student-Produced Responses ONLY ANSWERS THAT ARE GRIDDED WILL BE SCORED. YOU WILL NOT RECEIVE CREDIT FOR ANYTHING WRITTEN IN THE BOXES. PLEASE DO NOT WRITE IN THIS AREA SERIAL # Page 6 31 32 33 34 35 36 37 38 39 40 21 22 23 24 25 26 27 28 29 30 11 12 13 14 15 16 17 18 19 20 1 2 3 4 5 6 7 8 9 10 SECTION 6 A B C D E A B C D E A B C D E A B C D E A B C D E A B C D E A B C D E A B C D E A B C D E A B C D E A B C D E A B C D E A B C D E A B C D E A B C D E A B C D E A B C D E A B C D E A B C D E A B C D E A B C D E A B C D E A B C D E A B C D E A B C D E A B C D E A B C D E A B C D E A B C D E A B C D E A B C D E A B C D E A B C D E A B C D E A B C D E A B C D E A B C D E A B C D E A B C D E A B C D E A B C D E A B C D E A B C D E A B C D E A B C D E A B C D E A B C D E A B C D E A B C D E A B C D E A B C D E A B C D E A B C D E A B C D E A B C D E A B C D E A B C D E A B C D E A B C D E A B C D E A B C D E A B C D E A B C D E A B C D E A B C D E A B C D E A B C D E A B C D E A B C D E A B C D E A B C D E A B C D E A B C D E A B C D E A B C D E A B C D E A B C D E A B C D E A B C D E A B C D E Quality Assurance Mark You must use a No. 2 pencil and marks must be complete.Do not use a mechanical pencil.It is very important that you fill in the entire circle darkly and completely. If you change your response, erase as completely as possible. Incomplete marks or erasures may affect your score. COMPLETE MARKEXAMPLES OF INCOMPLETE MARKS 1 2 3 4 5 6 7 8 9 1 2 3 4 5 6 7 8 9 1 2 3 4 5 6 7 8 9 1 2 3 4 5 6 7 8 9 ⁄ 0 . ⁄ . . 0 0 . 1 2 3 4 5 6 7 8 9 1 2 3 4 5 6 7 8 9 1 2 3 4 5 6 7 8 9 1 2 3 4 5 6 7 8 9 ⁄ 0 . ⁄ . . 0 0 . 1 2 3 4 5 6 7 8 9 1 2 3 4 5 6 7 8 9 1 2 3 4 5 6 7 8 9 1 2 3 4 5 6 7 8 9 ⁄ 0 . ⁄ . . 0 0 . 1 2 3 4 5 6 7 8 9 1 2 3 4 5 6 7 8 9 1 2 3 4 5 6 7 8 9 1 2 3 4 5 6 7 8 9 ⁄ 0 . ⁄ . . 0 0 . 1 2 3 4 5 6 7 8 9 1 2 3 4 5 6 7 8 9 1 2 3 4 5 6 7 8 9 1 2 3 4 5 6 7 8 9 ⁄ 0 . ⁄ . . 0 0 . 1 2 3 4 5 6 7 8 9 1 2 3 4 5 6 7 8 9 1 2 3 4 5 6 7 8 9 1 2 3 4 5 6 7 8 9 ⁄ 0 . ⁄ . . 0 0 . 1 2 3 4 5 6 7 8 9 1 2 3 4 5 6 7 8 9 1 2 3 4 5 6 7 8 9 1 2 3 4 5 6 7 8 9 ⁄ 0 . ⁄ . . 0 0 . 1 2 3 4 5 6 7 8 9 1 2 3 4 5 6 7 8 9 1 2 3 4 5 6 7 8 9 1 2 3 4 5 6 7 8 9 ⁄ 0 . ⁄ . . 0 0 . 1 2 3 4 5 6 7 8 9 1 2 3 4 5 6 7 8 9 1 2 3 4 5 6 7 8 9 1 2 3 4 5 6 7 8 9 ⁄ 0 . ⁄ . . 0 0 . 1 2 3 4 5 6 7 8 9 1 2 3 4 5 6 7 8 9 1 2 3 4 5 6 7 8 9 1 2 3 4 5 6 7 8 9 ⁄ 0 . ⁄ . . 0 0 . CAUTION A B C D A B C D CCCCCCCC AA Section 7, the equating section of this practice test, has been omitted.

SAT Preparation Booklet 44 Quality Assurance Mark Page 7 31 32 33 34 35 36 37 38 39 40 21 22 23 24 25 26 27 28 29 30 11 12 13 14 15 16 17 18 19 20 1 2 3 4 5 6 7 8 9 10 SECTION 9 31 32 33 34 35 36 37 38 39 40 21 22 23 24 25 26 27 28 29 30 11 12 13 14 15 16 17 18 19 20 1 2 3 4 5 6 7 8 9 10 SECTION 10 31 32 33 34 35 36 37 38 39 40 21 22 23 24 25 26 27 28 29 30 11 12 13 14 15 16 17 18 19 20 1 2 3 4 5 6 7 8 9 10 SECTION 8 A B C D E A B C D E A B C D E A B C D E A B C D E A B C D E A B C D E A B C D E A B C D E A B C D E A B C D E A B C D E A B C D E A B C D E A B C D E A B C D E A B C D E A B C D E A B C D E A B C D E A B C D E A B C D E A B C D E A B C D E A B C D E A B C D E A B C D E A B C D E A B C D E A B C D E A B C D E A B C D E A B C D E A B C D E A B C D E A B C D E A B C D E A B C D E A B C D E A B C D E A B C D E A B C D E A B C D E A B C D E A B C D E A B C D E A B C D E A B C D E A B C D E A B C D E A B C D E A B C D E A B C D E A B C D E A B C D E A B C D E A B C D E A B C D E A B C D E A B C D E A B C D E A B C D E A B C D E A B C D E A B C D E A B C D E A B C D E A B C D E A B C D E A B C D E A B C D E A B C D E A B C D E A B C D E A B C D E A B C D E A B C D E A B C D E A B C D E A B C D E A B C D E A B C D E A B C D E A B C D E A B C D E A B C D E A B C D E A B C D E A B C D E A B C D E A B C D E A B C D E A B C D E A B C D E A B C D E A B C D E A B C D E A B C D E A B C D E A B C D E A B C D E A B C D E A B C D E A B C D E A B C D E A B C D E A B C D E A B C D E A B C D E A B C D E A B C D E A B C D E A B C D E A B C D E A B C D E A B C D E A B C D E A B C D E A B C D E A B C D E You must use a No. 2 pencil and marks must be complete.Do not use a mechanical pencil.It is very important that you fill in the entire circle darkly and completely. If you change your response, erase as completely as possible. Incomplete marks or erasures may affect your score. COMPLETE MARKEXAMPLES OF INCOMPLETE MARKS A B C D A B C D CCCCCCCC AA

Official SAT Practice Test 45 8 FORM CODE(Copy and gridason back of test book.) GMCM • • • • 514 • • • 9 TEST FORM (Copy from back of test book.) 06 1 35 14 YOUR NAME (PRINT) LAST FIRST MI TEST CENTER NUMBER NAME OF TEST CENTER ROOM NUMBER SAT Reasoning Test — General Directions Timing•Yo u will have 3 hours and 45 minutes to work on this test. •There are ten separately timed sections: One 25-minute essay Six other 25-minute sections Two 20-minute sections One 10-minute section •You may work on only one section at a time. •The supervisor will tell you when to begin and end each section. •If you finish a section before timeis called, check your work on that section. You may NOT turn to any other section. •Work as rapidly as you can without losing accuracy.Don’t waste time on questions that seem too difficult for you. Marking Answers•Be sure to mark your answer sheet properly. •You must use a No.2 pencil. •Carefully mark only one answer for each question. •Make sure you fill the entire circle darkly and completely. •Do not make any stray marks on your answer sheet. •If you erase, do so completely.Incomplete erasures may be scored as intended answers. •Use only the answer spaces that correspond to the question numbers. Using Your Test Book •You may use the test book for scratchwork, but you will not receive credit for anything written there. •After time has been called, you may not transfer answers to your answer sheet or fillincircles. •You may not fold or remove pages or portions of a page from this book, or take the book or answer sheet from the testing room. Scoring •For each correct answer, you receive one point. •For questions you omit, you receive no points. •For a wrong answer to a multiple-choice question, you lose one-fourth of apoint. If you can eliminate one or more of the answer choices as wrong, youincrease your chances of choosing the correct answer and earning one point. If you can’t eliminate any choice, move on.You can return to the question laterif thereistime. •For a wrong answer to a student-produced response (“grid-in”) math question, you don’t lose any points. •Multiple-choice and student-produced response questions are machine scored. •The essayis scored ona1to6scale by two different readers.The total essay scoreis the sum of the two readers’ scores. •Off-topic essays, blank essays, and essays writteninink will receive a score of zero. •If your essay does not reflect your original andindividual work, your test scores may be canceled. The passages for this test have been adapted from published material. Theideas containedin them do not necessarily represent the opinions of the College Board. IMPORTANT: The codes below are unique to your test book. Copy them on your answer sheet in boxes 8 and 9 and fill in the corresponding circles exactly as shown. DO NOT OPEN THIS BOOK UNTIL THE SUPERVISOR TELLS YOU TO DO SO.

SAT Preparation Booklet 46 The essay gives you an opportunity to show how effectively you can develop and express ideas. You should, therefore, take care to develop your point of view, present your ideas logically and clearly, and use language precisely. Your essay must be written on the lines provided on your answer sheet — you will receive no other paper on which to write. You will have enough space if you write on every line, avoid wide margins, and keep your handwriting to a reasonable size. Remember that people who are not familiar with your handwriting will read what you write. Try to write or print so that what you are writing is legible to those readers. Important Reminders: xDo not write your essay in your test book. You will receive credit only for what you write on your answer sheet. xIf your essay does not reflect your original and individual work, your test scores may be canceled. You have twenty-five minutes to write an essay on the topic assigned below. Given the importance of human creativity, one would think it should have a high priority among our concerns. But if we look at the reality, we see a different picture. Basic scientific research is minimized in favor of immediate practical applications. The arts are increasingly seen as dispensable luxuries. Yet as competition heats up around the globe, exactly the opposite strategy is needed. Invention studies, experience, or observations. If you finish before time is called, you may check your work on this section only. Do not turn to any other section in the test. Turn to page 2 of your answer sheet to write your ESSAY. Is creativity needed more than ever in the world today? Plan and write an essay in which you develop your ESSAY xA pencil is required for the essay. An essay written in ink will receive a score of zero. Think carefully about the issue presented in the following excerpt and the assignment below. Adapted from Mihaly Csikszentmihalyi, Creativity: Flow and the Psychology of Discovery and Assignment:Time — 25 minutes xAn off-topic essay will receive a score of zero. BEGIN WRITING YOUR ESSAY ON PAGE 2 OF THE ANSWER SHEET. point of view on this issue. Support your position with reasoning and examples taken from your reading,

Official SAT Practice Test 47 SECTION 2 Time — 25 minutes 18 Questions Turn to Section 2 (page 4) of your answer sheet to answer the questions in this section. Directions: This section contains two types of questions. You have 25 minutes to complete both types. For questions 1-8, solve each problem and decide which is the best of the choices given. Fill in the corresponding circle on the answer sheet. You may use any available space for scratchwork. 1. If 4319tu++= , then tu+= (A) 3 (B) 4 (C) 5 (D) 6 (E) 7 2. In the figure above, three lines intersect at a point. If f=85 and c=25, what is the value of a ? (A) 60 (B) 65 (C) 70 (D) 75 (E) 85

SAT Preparation Booklet 48 3. If Marisa drove n miles in t hours, which of the following represents her average speed, in miles per hour? (A) n t (B) t n (C) 1 nt (D) nt (E) nt2 4. Ifa is an odd integer and b is an even integer, which of the following is an odd integer? (A) 3b (B) a+3 (C) 2ab+ (D)ab+2 (E) 2ab+ 5. In the coordinate plane, the points F 21,, G14,, andH41, lie on a circle with center P. What are the coordinates of point P ? (A) 0 0, (B) 1 1, (C) 12, (D) 12, (E) 25 25.,. 6. The graph of ( )yfx is shown above. If 36,x for how many values of x does () 2? fx (A) None (B) One (C) Two (D) Three (E) More than three 7. If the average (arithmetic mean) of t and t+2 is x and if the average of t and t−2 is y, what is the average of x and y ? (A) 1 (B) t 2 (C) t (D) t+1 2 (E) 2t 8. For all numbers x and y, let xy  be defined as xy x xy y 22 . What is the value of ( )? 31 1 (A) 5 (B) 13 (C) 27 (D) 170 (E) 183

Official SAT Practice Test 49 9. Morgan’s plant grew from 42 centimeters to 57 centimeters in a year. Linda’s plant, which was 59 centimeters at the beginning of the year, grew twice as many centimeters as Morgan’s plant did during the same year. How tall, in centimeters, was Linda’s plant at the end of the year? 10. Since the beginning of 1990, the number of squirrels in a certain wooded area has tripled during every 3-year period of time. If there were 5,400 squirrels in the wooded area at the beginning of 1999, how many squirrels were in the wooded area at the beginning of 1990 ?

SAT Preparation Booklet 50 11. In the figure above, triangles ABC and CDE are equilateral and line segment AE has length 25. What is the sum of the perimeters of the two triangles? 12. Marbles are to be removed from a jar that contains 12 red marbles and 12 black marbles. What is the least number of marbles that could be removed so that the ratio of red marbles to black marbles left in the jar will be 4 to 3 ? x v vt xpt= = =3 4 13. For the system of equations above, if x 0, what is the value of p ? 14. If 211,x what is one possible value of x ?

. . . . . .